Princeton Review exams

अब Quizwiz के साथ अपने होमवर्क और परीक्षाओं को एस करें!

Which one of the following is true for all liquids? A. Liquids are always more stable than gases or solids. B. A higher vapor pressure indicates a lower heat of vaporization. C. As the pressure on a liquid increases, the temperature of that liquid decreases. D. Fluid pressure is equal at all points in a liquid.

B. Statements A, C, and D are all false, so the answer must be B. The vapor pressure of a liquid is the pressure at which vaporization and condensation are at equilibrium. Higher vapor pressure implies weaker intermolecular forces, and, consequently, a lower heat of vaporization.

Antibodies specific to the mitotic spindle apparatus would most likely recognize products of: A. transcription. B. translation. C. transformation. D. replication.

B. The spindle is composed of microtubules, which are polymers of tubulin protein monomers. Antibodies against the spindle would recognize proteins, which are produced during translation (choice B is correct). Choices A, C, and D are wrong because these processes do not produce proteins.

A researcher bubbles excess oxygen through a solution containing Ag2S, waits five minutes, and concludes that no detectable reaction occurred. She leaves the reaction on her bench overnight, and returns in the morning to find the reaction complete. What can she conclude from these results? A. Additional oxygen from the atmosphere entered her sample and caused the reaction to go to completion. B. Sulfur is more reactive than oxygen. C. The ΔG of this reaction is positive. D. The activation energy barrier for the reaction of Ag2S with O2 is relatively high.

D. A lengthy reaction rate is a kinetic factor and only choice D addresses kinetics. Choice A is eliminated because excess oxygen was added at the start. Choice B is eliminated because the reaction would not go to completion if sulfur was more reactive than oxygen. Since the reaction eventually went to completion it must be spontaneous with a negative ΔG, so choice C is eliminated.

A school-aged boy has demonstrated an understanding of conservation, is focused on living up to the expectations of others, and tries to be a "good boy", but cannot yet reason about abstract moral concepts and does not yet feel a duty to uphold social rules and conventions. He is likely to be in which of the following stages, according to Piaget and Kohlberg, respectively? A. Preoperational thought; Stage 2 (self-interest orientation) B. Concrete operational thought; Stage 2 (self-interest orientation) Your Answer C. Preoperational thought; Stage 3 (interpersonal accord and conformity) D. Concrete operational thought; Stage 3 (interpersonal accord and conformity)

D. According to Piaget, a child who has demonstrated a grasp of conservation but cannot yet reason about abstract moral concepts should fall within the concrete operational thought stage; children in the preoperational thought stage have not yet grasped the principle of conservation (choices A and C can be eliminated). According to Kohlberg, a child who tries to live up to the expectations of others and to be a "good boy", but does not yet feel a duty to uphold social rules and conventions, should fall within the third stage of moral reasoning (choice B is wrong and choice D is correct).

The Ka for the first ionization of sulfurous acid (1.7 × 10-2) is significantly larger than the Ka for the second ionization (6.4 × 10-8). A likely explanation for this is that: A. the electronegativity of the remaining hydrogen atom increases after the first hydrogen ion has been removed. B. the second ionization can only take place if the first ionization proceeds to completion. C. neutral hydrogen is difficult to ionize in aqueous solution. D. the remaining hydrogen atom experiences greater electrostatic attraction after the loss of the first hydrogen ion.

D. After loss of the first proton, the remaining hydrogen is bound to a negatively-charged molecule. Electrostatic attraction between this remaining hydrogen and the negatively-charged molecule would disfavor loss of the second proton, resulting in a smaller K. Therefore, K2 << K1.

The capacity of working memory is about ________ items.

7+/-2

A patient presents to the hospital after prolonged vomiting and exhibits metabolic alkalosis, a loss of hydrogen ions in the bodily fluids. To buffer the pH change, the patient's body begins to: A. hypoventilate to increase Partial pressure of Carbon dioxide or P subscript CO subscript 2. B. hyperventilate to decrease Partial pressure of Carbon dioxide or P subscript CO subscript 2. C. release insulin to decrease glucose blood level. D. renally eliminate titratable acids to increase hydrogen ion concentration.

A. A decreased hydrogen ion concentration corresponds to an increased pH. In order to buffer the pH change, the body will try to decrease the pH. According to Equation 1, an increase in the partial pressure of carbon dioxide would lower the pH (eliminate choice B, choice A is correct). Glucose and insulin are not mentioned anywhere in the passage (eliminate choice C). Renally eliminating acids would further decrease the hydrogen ion concentration (eliminate choice D).

A 42 year-old female presents with hemolytic anemia (inappropriate destruction of red blood cells) secondary to a pyruvate kinase deficiency (the final enzyme in glycolysis). What best describes the mechanism of her hemolytic anemia? A. Inadequate ATP resulting in failure of primary active transport and subsequent cell damage B. Inadequate ATP resulting in failure of secondary active transport and subsequent cell damage C. Increased Krebs cycle activity to compensate for the glycolytic blockage D. Depletion of intracellular glucose leading to apoptosis

A. A deficiency in the final enzyme of glycolysis would significantly reduce ATP production and in red blood cells, which lack mitochondria, glycolysis is the principal source of ATP. Pumps relying on ATP, primary active transporters, would fail leading to cell damage; the damaged cells would then ultimately be consumed by a macrophage in the spleen (choice A is correct and B is wrong). Without mitochondria, red blood cells do not utilize the Krebs cycle and inability to perform glycolysis would lead to glucose accumulation, not depletion (choices C and D are wrong).

In the semantic network model, what determines the strength of a connection between a node and an association? A. How frequently and deeply connections are made B. How closely the node and association are explicitly related C. How many links there are between the node and the association D. How many state-dependent cues have been created between the node and the association

A. According to the semantic network model, the strength of a connection between a node and an association is related to how frequently and how deeply the connections are made (choice A is correct). The explicit relationship between the node and the association is meaningless unless that connection is constantly reinforced (choice B is wrong). Separate links between the node and the association—suggesting a more circuitous route—are indicative of the spreading activation pattern (choice C is wrong). State-dependent cues, in which familiar locales are used to trigger memories, do not determine the strength of the connection between a node and an association unless, once again, they are used with great frequency (choice D is wrong).

In which of the following are the acrosomal enzymes synthesized? A. Rough endoplasmic reticulum B. Cytoplasm C. Golgi apparatus D. The nucleus, since sperm have no cytoplasm

A. Acrosomal enzymes are located inside a membrane-bound vesicle and are going to be secreted. It is likely that they are processed through the secretory pathway, and to do so they must be synthesized in the rough ER (choice A is correct), not the cytoplasm (choice B is wrong). Proteins are never synthesized in the Golgi (choice C is wrong). It is true that sperm have very little cytoplasm, but during spermatogenesis they did originally have all normal cell structures. Also, protein synthesis never occurs in the nucleus (choice D is wrong).

What energy does ATP synthase use directly in order to catalyze the formation of ATP from ADP and inorganic phosphate? A. The energy of the proton gradient generated by the electron transport chain B. The energy released by the hydrolysis of GTP C. The energy released via coupling of ATP synthase to substrate-level phosphorylation D. The energy of the conformational change induced by binding of ADP and inorganic phosphate to ATP synthase's catalytic domain

A. After being pumped across the inner mitochondrial membrane, protons re-enter the mitochondrial matrix. They do so by flowing through a specific channel in the F0 subunit of ATP synthase, down their chemiosmotic gradient. This proton flow leads to a conformational change and rotation of the F1 domain, driving ATP synthesis (choice A is true and is the correct answer). ATP synthase's action is not coupled to hydrolysis of GTP nor to substrate-level phosphorylation (choices B and C are false and may be eliminated). Linking of ADP and inorganic phosphate is induced by a conformational change that harnesses the energy of the chemiosmotic gradient described (choice D is false and may be eliminated).

Stearic acid (or octadecanoic acid) is a saturated fatty acid with the formula CH3(CH2)16COOH After undergoing β-oxidation, it will generate: A. 9 acetyl-CoA, 9 NADH and 9 FADH2. B. 9 acetyl-CoA, 8 NADH and 8 FADH2. C. 8 acetyl-CoA, 8 NADH and 8 FADH2. D. 8 acetyl-CoA, 7 NADH and 7 FADH2.

B. Stearic acid has 18 carbons and so will undergo 8 rounds of β-oxidation. This will generate 9 molecules of acetyl-CoA (eliminate choices C and D), 8 molecules of NADH and 8 molecules of FADH2 (choice B is correct, eliminate choice A).

Multiple sclerosis (MS) is an autoimmune disease that results in demyelination of central nervous system axons. Which of the following could change as a result of this condition? I. Perception of sensory stimuli II.Temporal summation in a postsynaptic neuron III.The strength of an action potential in a demyelinated axon

Items I and II are true: Demyelination of an axon can result in slowed or lost conduction of action potentials, which could affect all types of stimuli perceived by the nervous system (choice B can be eliminated). Specifically, this affects the timing of neurotransmitter release by the neuron, which then changes the frequency of EPSPs or IPSPs in the postsynaptic cell (choice A can be eliminated). Item III is false: Action potentials are "all-or-none" phenomena, and their magnitude does not change (choice C is correct and choice D is wrong).

The process of culturing bacteria often involves inoculation of cells on a noncellular, agar-based medium. Such a methodology would NOT result in growth of animal viruses because animal viruses: A.are obligate parasites. B.lack DNA. C.assimilate carbon. D.require essential vitamin supplements for growth.

The answer is A because viruses can only reproduce in a host cell, and are therefore obligate intracellular parasites.

If the GAPDH gene is continuously expressed, where is it most likely found? A.In euchromatin B.In a telomere C.In heterochromatin D.In a centromere

The answer to this question is A because GAPDH is a housekeeping gene and is expressed continuously. A gene that is always turned on must be accessible to transcription factors. Only euchromatin is in a loose conformation and readily accessible for transcription.

Which statement correctly describes how enzymes affect chemical reactions? Stabilization of: a) the substrate changes the free energy of the reaction b) the transition state changes the free energy of the reaction c) the transition state changes the activation energy of the reaction d) c) the substrate changes the activation energy of the reaction

The correct answer is D. This is a Biochemistry question that falls under the content category "Principles of chemical thermodynamics and kinetics." The answer is D because the stabilization of the transition state, not the substrate, provides binding energy that is used to lower the activation energy. It is a Knowledge of Scientific Concepts and Principles question because you are asked to recognize a fundamental principle of enzyme kinetics.

Which of the following will decrease the percentage ionization of 1.0 M acetic acid, CH3CO2H(aq)? A.Chlorinating the CH3 group B.Diluting the solution C.Adding concentrated HCl(aq) D.Adding a drop of basic indicator

c- because Hal is a strong acid that will increase the amount of H+ in solution and thus decrease the percentage of CH3CO2H that ionizes. A strong acid to a solution of weak acid will decrease the amount of ionization of the latter.

A homodimeric protein was found to migrate through SDS polyacrylamide gel electrophoresis (SDS-PAGE) with a mobility that matched that of a 45-kDa standard. What change in the experiment would increase the chances of observing the mobility expected for the 22.5-kDa monomer? A.Increasing the gel running time B.Adding a reducing agent C.Using a higher voltage D.Removing the SDS

the answer to this question is B because adding a reducing agent would eliminate any disulfide bridges and allow the monomers to run separately-thus leading to a migration expected for the 22.5-kDa protein

Which personality disorder describes someone who has a history of unstable relationships, a diffuse and unreliable sense of identity, chronic feelings of emptiness, and a history of suicidal thoughts and gestures? A. Schizotypal personality disorder B. Dependent personality disorder C. Borderline personality disorder D. Antisocial personality disorder

time; negative values indicate decreases) This image is too complex to describe in a useful or meaningful way. Figure 2 Differences between men's and women's personality changes in their reactions to a specific major life event Adapted from J. Specht, B. Egloff, & S. Schmukle. Stability and change of personality across the life course: The impact of age and major life events on mean-level and rank-order stability of the Big Five. © 2011 by American Psychological Association Question 37 Which personality disorder describes someone who has a history of unstable relationships, a diffuse and unreliable sense of identity, chronic feelings of emptiness, and a history of suicidal thoughts and gestures? A. Schizotypal personality disorder B. Dependent personality disorder C. Borderline personality disorder Correct Answer D. Antisocial personality disorder

Conduction aphasia occurs when some sort of damage occurs to the connections between Wernicke's area and Broca's area in the brain. Although language comprehension and production are both individually intact, this disconnect leads to specific problems with: A. repeating information back to someone. B. reading printed words. C. writing. D. facial recognition.

A. Although a patient with conduction aphasia will understand what is being said to him or her and can communicate back, this disconnect caused by damage to the arcuate fasciculus makes it particularly hard to repeat aloud what has been said to him or her; the information cannot be relayed from Wernicke's area (processing) to Broca's area (production) properly (choice A is correct). Reading skills may still be intact, as the Wernicke's area is not damaged (choice B is wrong), and writing skills may also be intact, as Broca's area is not damaged (choice C is wrong). An inability to recognize faces, known as prosopagnosia, is due to damage to a certain visual pathway in the occipital and temporal lobes and is not related to language capabilities (choice D is wrong).

Similarities that exist between protists and bacteria include all of the following except: A. Organelles B. Double-stranded DNA C. Unicellularity D. Flagellar motility

A. Bacteria do not have organelles while protists exhibit the organelles typical of animal cells (answer choice A is correct). While bacterial store genetic information in circular DNA and protists have linear chromosomes, both forms of DNA are double-stranded and, thus, this is a similarity between bacteria and protists (answer choice B is incorrect). Bacteria are unicellular and protists typically are, as well (answer choice C is wrong). Some protists exhibit amoeboid movement but both protists and bacteria often exhibit flagellar movement (answer choice D is wrong).

Psychologist Ernest Hilgard also replicated the ice bath study and found that, although participants reported that they felt no pain, when asked to hit a button if some part of them felt pain, many did. Which psychological phenomenon might explain this behavior? A. Dissociation B. Accommodation C. Conformity D. Extinction

A. Dissociation is defined as a split in consciousness, and this phenomenon can allow contradictory thoughts or behaviors to occur simultaneously. In the case of a person reporting no pain but indicating otherwise, dissociation might explain this behavior (choice A is correct). Accommodation is the process by which one adapts one's current understanding to incorporate new information; the stem does not provide any information to suggest that accommodation was relevant (choice B is wrong). Conformity refers to the phenomenon in which individuals adjust their behaviors or beliefs to coincide with those of the dominant group; there is no information in the stem to suggest that individuals were under pressure to conform (choice C is wrong). Extinction is a term used in classical and operant conditioning and refers to the diminishment of a conditioned association; there is no evidence of conditioning in the question stem (choice D is wrong).

(+)-Ginkgolide B is a diterpenoid trilactone with six five-membered rings, and is extracted from the root bark and leaves of the Ginkgo biloba tree. This molecule has blood-brain barrier permeability, the molecular formula C20H24O10 and a molecular weight of 424.4 g/mol. This molecule must contain: A. four isoprene units, additional functional groups and is able to perform simple diffusion through endothelial cells connected by tight junctions B. four isoprene units, no additional functional groups and is able to perform simple diffusion through endothelial cells connected by tight junctions. C. four terpene units, additional functional groups and is able to perform simple diffusion through neurons connected by tight junctions. D. four isoprene units, additional functional groups and is able to perform simple diffusion through endothelial cells connected by gap junctions.

A. Diterpenoids contain four isoprene units (eliminate choice C) and additional functional groups (eliminate choice B). The blood-brain barrier is a highly selective permeability barrier that separates the circulating blood from the central nervous system. It is formed by endothelial cells (another reason to eliminate choice C), which are connected by tight junctions (eliminate choice D). Choice A is correct because it accurately describes the composition of the molecule.

What did Emile Durkheim believe about crime? A. A society without crime is impossible. B. As sociologists continue to learn more about crime, they will eventually be able to suggest social changes that will help eliminate crime. C. Crime is essentially abnormal. D. All forms of crime are normal.

A. Durkheim believed that crime is present in every society; it cannot, and should not, be eliminated (choice A is correct). According to Durkheim, regardless of how much sociologists learn, crime will not disappear (choice B is wrong). Durkheim viewed crime as being a normal part of society (choice C is wrong). Although he viewed crime as basically normal, Durkheim acknowledged that some forms of crime can be abnormal (choice D is wrong).

Which one of the following processes does NOT take place in the mitochondrion? A. Fatty acid synthesis B. Krebs cycle C. Electron transport D. Oxidation of pyruvic acid

A. Fatty acid synthesis occurs in the cytosol (choice A is correct). The Krebs cycle, electron transport, and the oxidation of pyruvic acid all happen in the mitochondrion (choices B, C, and D are wrong).

TUNEL (terminal deoxynucleotidyl transferase dUTP nick end labeling) is a method for detecting DNA fragmentation by labeling the terminal end of nucleic acids. BrdU (5-bromo-2'-deoxyuridine) can also be used in staining protocols, as this synthetic nucleoside can be incorporated into nucleic acids during replication instead of thymidine. Tumor cells have: A. minimal TUNEL staining due to their low apoptotic rate, but strong BrdU staining due to active genome replication. B. strong TUNEL staining due to their high apoptotic rate, and strong BrdU staining due to active genome replication. C. strong TUNEL staining due to their high apoptotic rate, but weak BrdU staining due to minimal genome replication. D. minimal TUNEL staining due to their low apoptotic rate, and weak BrdU staining due to minimal genome replication.

A. Genome fragmentation is one of the hallmarks of apoptosis and tumor cells generally evade apoptosis. This means tumor cells will have minimal TUNEL staining (eliminate choices B and C). Tumor cells have high proliferation rates and so will be actively replicating the genome. This means they will likely display strong BrdU staining (choice A is correct, eliminate choice D).

If the plasma glucose falls below normal, which of the following conditions is true regarding pancreatic activity? A. High levels of glucagon will be produced from alpha cells. B. High levels of insulin will be produced from beta cells. C. Pancreatic acinar cells will secrete high levels of somatostatin. D. The pancreas will produce high levels of both insulin and glucagon.

A. Glucagon is secreted by alpha cells in the pancreas and acts to elevate plasma glucose if it falls too low (choice A is correct). Insulin acts to lower blood glucose and will not be secreted in response to low blood glucose (choices B and D are wrong).Somatostatin is secreted in response to raised glucose or other nutrients and acts to repress digestive functions, so it will not be elevated in response to decreased glucose (choice C is wrong).

In humans, the fusion of the plasma membranes of a sperm and an ovum is followed first by which of the following? A. Release of the second polar body from the fertilized ovum B. Implantation in the uterus C. The first cell division of the zygote D. Gastrulation

A. Human ova do not complete the second meiotic cell division, including the formation of the second polar body, until after fertilization (choice A is correct). This process occurs immediately after fertilization and is a prerequisite for the first cell division of the zygote (choice C is wrong). Implantation happens a few days later, and gastrulation occurs at a stage many cell divisions after the first cell division (choices B and D are wrong).

How might a theorist in the tradition of symbolic interactionism explain deviance? A. Deviant behavior is a response to being labeled a deviant by others. B. Deviant behavior results from performing on the back stage when one should be performing on the front stage. C. Deviance is a failure to develop a looking-glass self. D. Deviance is a failure to resolve the dilemma of identity versus role confusion in adolescence.

A. Labeling theory is a fundamental concept of symbolic interactionism. It states that deviance is not inherent in the act itself, but is rather an attribution from others (such as police, family, or clergy; choice A is correct). Back stage and front stage dynamics are an aspect of the dramaturgical approach. If one acts on the back stage when he or she should be performing on the front stage, the audience might experience more than it otherwise would (e.g., more details about a person's motivations or inner thoughts), but this would not necessarily be a deviant act (choice B is wrong). A looking-glass self describes how people shape their self-concepts based on how they think others perceive them (choice C is wrong). The dilemma of identity versus role confusion is one of Erikson's eight developmental stages; it is not an aspect of symbolic interactionism (choice D is wrong).

If lactic acid radiolabeled with carbon-11 were injected intramuscularly in a patient, where would radioactive carbon most likely be detected? A. Expired breath B. Sweat C. Feces D. Hydroxyapatite

A. Lactic acid, which dissociates to become lactate at physiological pH, is converted to acetyl-CoA in the liver before rejoining cellular respiration and ultimately being released as carbon dioxide (choice A is correct). Any quantity of organic molecules excreted into the sweat and feces will be much smaller (choice B and C are wrong) and hydroxyapatite, the principal component of bone, does not contain carbon (choice D is wrong).

According to Lawrence Kohlberg, what is the highest level of morality that the average adult tends to achieve? A. She or he will acknowledge a sense of duty to uphold the laws of the land and maintain basic social conventions. B. He or she will strive for social justice—the greatest good for the greatest number. C. He or she will focus on the approval (or disapproval) of others, acting in accordance with the views of his or her community or peer group. D. She or he will act in such a way as to avoid punishment at all costs.

A. Lawrence Kohlberg postulated six stages of moral development, but said that the average person generally does not pass beyond the fourth stage, in which he or she acknowledges a sense of duty to uphold the law and maintain social conventions (choice A is correct). Striving for social justice, wherein one acts for the greatest good for the greatest number, is the fifth stage in Kohlberg's schema and one that he proposed that few people reach (choice B is wrong). A focus on receiving approval from one's peers along with a need to conform is the third stage of development, usually seen in older children and adolescents (choice C is wrong). Acting to avoid punishment is the earliest stage of moral development, typical of very young children (choice D is wrong).

Object permanence allows human beings to realize that an object continues to exist outside of their perception of it. During which of Jean Piaget's stages does this ability develop? A. Sensorimotor B. Preoperational C. Concrete operational D. Formal operational

A. Object permanence develops during Piaget's first stage, the sensorimotor stage (choice A is correct). During all of the subsequent stages, according to Piaget's theory of cognitive development, children are using the object permanence skill that they previously developed. During the preoperational stage, children think more symbolically as they develop memory and imagination (choice B is wrong). Next is the concrete operational stage, when children begin to exhibit logical reasoning (choice C is wrong). This is followed by Piaget's final stage, that of formal operations, when children are able to consider abstract relationships and concepts (choice D is wrong).

Contraction of cardiac muscle begins just after depolarization and lasts about as long as the action potential. During a contraction: A. Ca2+ enters the cell through voltage-gated ion channels. B. sarcomere length increases. C. polymerized actin becomes depolymerized. D. myosin binds irreversibly to actin.

A. Part of the cardiac action potential is the opening of slow voltage-gated calcium channels to create the plateau in depolarization (Phase 2) so choice A is correct. When the voltage-gated calcium channels open, calcium enters the cell and can play a role in the contraction of the cardiac muscle. Sarcomere length decreases during each contraction, and does not increase (choice B is wrong). Actin filaments in contractile tissue are not dynamic as they are in other processes and do not spontaneously depolymerize and repolymerize (choice C is wrong). Myosin binds actin reversibly, not irreversibly (choice D is wrong).

Which of the following salts, when dissolved in water, would produce the solution with the highest pH? A. Na2CO3 B. NH4Cl C. NaCl D. AlBr3

A. The most basic salt will produce the solution with the highest pH. The carbonate ion from Na2CO3 is the conjugate base of bicarbonate, which is itself weakly basic in aqueous solution; choice A is the most basic salt listed. NaCl is a neutral salt as neither of its ions are reactive with water (eliminate choice C). Both NH4Cl and AlBr3 are acidic salts because both cations will interact with water to increase the [H+] in solution (eliminate choices B and D).

Media representations of Planned Parenthood often center on their abortion practices. Planned Parenthood is responsible for an annual total of 300,000 abortions. However, this accounts for a mere three percent of the services provided. This misrepresentation of Planned Parenthood is an example of: A. institutional discrimination. B. the McDonaldization of society. C. a coercive organization. D. symbolic interactionism.

A. Planned Parenthood experiences the effects of prejudice as a result of practices, such as abortion, that conflict with the beliefs of some groups, and this prejudice, in some cases, contributes to institutional discrimination. Mass media is a powerful institution in the United States, and the media executives with higher socioeconomic status are able to set the agenda for their programs. This can lead to institutional discrimination in situations when the media presents a specific group in a negative light for its own purposes (e.g., the anti-abortion movement; choice A is correct). The McDonaldization of society described when a culture adopts the characteristics of a fast-food restaurant, such as efficiency, calculability, standardization, and control. While aspects of mass media are certainly a good example of McDonaldization, the misrepresentation of Planned Parenthood by the media is not an example of this (choice B is wrong). A coercive organization is one where members have no choice in joining; for example, a prison is an example of a coercive organization. While Planned Parenthood has little choice in what the media portrays about them, as an organization, Planned Parenthood is not a part of the media - these are two separate organizations. Therefore, this is not an example of a coercive organization (choice C is wrong). Symbolic interactionism is a major sociological theory that suggests that people create symbolic meanings about objects, events, and behaviors. For example, a handshake is a symbol in society that conveys many different meanings—and everyone in society pretty much acts toward this symbol in agreed-upon ways. Symbolic interactionism is a micro-level theory of society, which focuses on interactions between individuals. Therefore, even if the interaction between the media and Planned Parenthood neatly conveyed something symbolic, it still could not be an example of symbolic interactionism because these are two social structures, which is a macro-level view of society (choice D is wrong).

Suppose that parents catch their teenage daughter stealing beer from a convenience store. If the parents want to decrease the likelihood that their daughter will steal again, which of the following actions would constitute a positive punishment to this desired end? A. Harshly rebuking their daughter upon detection of the stolen beer B. Denying their daughter television, Internet, and smart phone privileges for one week C. Forcing their daughter to return the beer to the store and to write the store owner a letter of apology D. Buying their daughter a stash of her favorite soft drinks for her to drink in lieu of beer

A. Positive punishment involves applying an undesirable stimulus after a given behavior with the intent of decreasing the likelihood that the behavior (stealing) will happen again. Harshly rebuking their daughter involves the parents' applying a negative stimulus, and thus constitutes positive punishment (choice A is correct). Denying the daughter that which she enjoys (television, Internet, and smart phone) would be a form of negative punishment, which involves removing desirable stimuli as a consequence of the behavior (stealing) with the intent of decreasing the likelihood that the behavior will occur again (choice B is wrong). Forcing their daughter to return the beer might also constitute a form of negative punishment (since the beer is likely a desirable stimulus), but writing the apology letter is at best an unclear consequence and is certainly not a form of positive punishment (choice C is wrong). Buying the daughter a stash of her favorite soft drinks in response to her stealing beer would constitute positive reinforcement, which involves applying a desirable stimulus after a given behavior in order to increase, not decrease, the likelihood that the behavior at issue will occur again. Thus, reacting to the theft in this way might very well have the unintended consequence of rewarding the undesirable behavior, thus inadvertenty causing the daughter to steal again (choice D is wrong).

DNA "melting" is the process by which double-stranded DNA (dsDNA) is separated into single-stranded DNA (ssDNA). Which of the following accounts for an increase in the UV absorption of a sample of DNA undergoing DNA melting? The decreased capacity of ssDNA for H-bonding Structural changes altering conjugation across the aromatic DNA bases Greater functional group vibration at high temperature A. II only B. III only C. II and III only D. I, II, and III

A. Process of elimination is the most effective method of determining the correct answer choice in this Roman numeral question. Item I is false: since the two strands of dsDNA are held together by networks of hydrogen bonds, once separated, the ssDNA will have many new, available sites for H-bonding (eliminate choice D). Item II is true: UV-Vis spectroscopy is used to measure changes in light absorption in highly conjugated systems, such as in aromatic compounds. The aromatic bases in dsDNA are tightly held in a coiled arrangement by H-bonds. When these interactions are removed in DNA melting, the chains can structurally relax. This allows changes in the conjugation of the bases, altering how much light is absorbed by a sample (eliminate choice B). Item III is false: changes in the vibrations of functional groups are monitored by infrared radiation, as UV radiation is far too energetic to induce such transitions (eliminate choice C).

RNA polymerase differs from DNA polymerase III in that RNA polymerase: does not require a primer. lacks exonuclease activity. travels in the 5' to 3' direction on the template strand. A. I and II only B. I and III only C. II and III only D. I, II, and III

A. RNA polymerase does not require a primer (statement I is true), and it does not have exonuclease activity (statement II is also true). RNA polymerase synthesizes RNA in the 5' to 3' direction, which means it must travel 3' to 5' on the template strand (statement III is false).

1-Methylcyclohexanol reacts with HBr to form 1-bromo-1-methylcyclohexane. The mechanism for this reaction is likely to be an: A. SN1 reaction. Correct Answer B. SN2 reaction. C. Nucleophilic addition. D. Addition-elimination.

A. Since no double bonds are formed or broken, this must be a substitution reaction (eliminate choices C and D). Since the hydroxyl in the starting material is on a tertiary carbon atom, the mechanism cannot occur by a bimolecular pathway (eliminate choice B.) The protonated OH group (under acidic conditions) will leave as water to yield a tertiary carbocation, which will be attacked by the bromide ion to give the product.

Addition of which of the following to a 100 mL solution of aqueous HI at pH = 1 would result in a new solution with a pH closest to 7? A. 230 mg of solid Na B. 100 mL of a 1 M solution of Ni(NO3)2 C. 420 mg of solid NaF D. 100 mL of a 0.01 M solution suspension of Zn0

A. The H+ in the HI solution can be eliminated electrochemically or through an acid base reaction to achieve a pH = 7. Neither the Ni2+ nor the Zn0 in choices B and D, respectively, have the capacity to reduce H+. Such a reaction with Zn0 has a negative voltage, and Ni2+ is simply not reductive. The NaF (MW = 42) in choice C does have the capacity to neutralize H+ after dissociating into Na+ and F-, and 420 mg of the compound does represent the correct 0.01 moles of compound needed to neutralize the 0.01 moles of acid in 100 mL of a 1 M solution. However, this constitutes adding a weak base to a strong acid, and will hence have a residual pH somewhere below 7. 230 mg of solid Na is 0.01 moles of Na. Since the reaction 2Na + 2H+ → H2 + 2Na+ is clearly electrochemically spontaneous, all of the H+ will be neutralized, leaving only a spectator ion in Na+. This makes choice A correct.

The amphoteric character of amino acids best explains their ability to: A. form dipolar ions. B. form peptide bonds. C. contribute directly to a protein's secondary structure. D. dissolve in nonpolar solvents.

A. The amphoteric character of amino acids describes their ability to do two things: accept a proton or donate a proton. In other words, amino acids can act as either an acid or a base. When the amino portion of an amino acid deprotonates its own carboxylic acid, a dipolar ion (or zwitterion) forms. This is a direct result of the amphoteric character of the amino acid.

Surgical severing of the corpus callosum is a potential treatment for severe epilepsy. A post-surgical epileptic is in the car, on the way home from the hospital. Which of the following would be expected when the person is looking straight ahead? A. The person will be able to verbalize the names of streets he sees on the right side of the road but not the names of streets on the left side of the road. B. The person will be able to verbalize the names of streets he sees on the left side of the road but not the names of streets on the right side of the road. C. The person will not be able to verbalize the names of streets on either side of the road. D. The person will be able to verbalize the names of streets on both sides of the road, but they will appear upside down.

A. The right side of the visual field of both eyes is processed on the left side of each retina. These images are then routed by axons to the left cerebral hemisphere. Similarly, the left side of the visual field is reflected onto the right side of each retina and routed to the right cerebral hemisphere. Assuming the corpus callosum (CC) is needed for the two hemispheres to share information, the absence of a CC would mean that images delivered to the left hemisphere remain in the left hemisphere, and images delivered to the right hemisphere remain in the right hemisphere. Since language is produced in the left hemisphere, the person would thus be able to verbalize images captured in the left hemisphere, which are those from the right side of the visual field (choice A is correct; choice C is wrong). They would not be able to verbalize those images captured in the right hemisphere, which are those from the left side of the visual field (choice B is wrong). Additionally, the CC is not responsible for correcting the inverted image as it is apprehended by the retina. Rather, that is the function of the visual cortex (choice D is wrong).

A soluble protein with a specific three-dimensional conformation is gently heated until it no longer displays activity. If at this point it is then allowed to cool, the protein resumes its function. The best explanation for these observations is that: A. the bonds that produce the three-dimensional configuration of a protein, once broken, are able to reform under certain conditions. B. the bonds that produce the three-dimensional configuration of a protein are impervious to changes in temperature. C. when a protein undergoes denaturation, the denaturation is always reversible. D. extremes in temperature will always produce reversible denaturation of a protein.

A. The three-dimensional structures of proteins are very important to their functions. Destroying the three dimensional structure by heating or otherwise denaturing the protein can thus destroy the protein's function. In some cases however, such as mild heating, the bonds are able to reform once the protein has cooled, and the protein function is restored (A is correct, and B is wrong). C and D are poor answer choices because of the word "always." Denaturation is not always reversible (C is wrong), and extremes in temperature often produce irreversible destruction (D is wrong). Also in the case described in the question, the protein was only mildly heated, implying that extremes in temperature were not reached.

One of the modifications that Milgram made to his original shock experiment was to test the possible effects of conformity on obedience. The basic experiment tests the conditions under which a research subject, the "teacher", will administer an electric shock to the "learner" (a confederate member of the research team). The teacher receives instructions from an authoritative "experimenter" (a research team member) to administer the shock under certain circumstances. Which of the following experimental modifications would in fact test for the effects of conformity on obedience? Adding another teacher (a confederate) to the shock administration room Adding a window to the room so that the teacher can see the learner Increasing the proximity of the teacher to the learner A. I only B. II only C. I and II only D. II and III only

A. To assess the possible effects of conformity on obedience, one must determine whether and to what extent subjects conform their behavior to the actions of others who appear to be similarly situated (i.e., other "teachers") when determinig whether or not to obey the experimenter who instructs them to administer shocks. Item I is true: adding another confederate teacher to the shock administration room would test whether the naive subject would conform to the actions of the other "teacher" when determining whether or not to obey. Item II is false: adding a window to the room wherein the learner will receive the shock would not test for the effects of conformity on obedience because it would not include a similarly situated figure (choices B and C can be eliminated). Item III is false: similarly, merely increasing the proximity of the teacher to the learner would only test obedience, as there is no other person present to whose behavior the subjects might conform their actions (choice D can be eliminated, choice A is correct).

Which of the following would be increased the most in an individual with uncontrolled diabetes? A. Acetyl-CoA B. Ketone bodies C. Lactic acid D. NADH

B. An individual with uncontrolled diabetes would have hyperglycemia and a relative (or complete) deficiency of insulin. This means that glucose levels would be elevated in the blood and unable to get inside of insulin-responsive tissues. Thus, any molecule that is produced mainly through glucose catabolism and cannot be made from alternative pathways (such as lactic acid), would be decreased, not increased (choice C is incorrect). In addition to being produced during cellular respiration, NADH can be made in the Krebs cycle and acetyl-CoA can be made from fatty acid oxidation would enter the Krebs cycle; however the greatest increase would be in the production of ketone bodies (choices A and D are eliminated, choice B is correct). Ketone bodies represent an alternate energy molecule in the absence of glucose.

Which of the following is NOT true regarding stereotype threat? A. Telling a group of females that "females tend to perform more poorly on this math test than males" before the math test results in lower average scores for the females. B. White males are unaffected by stereotype threat because they are the majority group to which other groups are often compared. C. Emphasizing students' race before a standardized test often produces lower scores for ethnic and racial minorities. D. Females tend to score higher on math-based tests when testing in a group of all females (rather than a mixed group of females and males).

B. Any group can be subject to stereotype threat, regardless of if the group is a majority or minority group. For example, when a group of white male students, who scored high in math on the SAT, were told that they were taking a test to "determine why Asian students typically outperform other students in math," they performed much worse in math. Stereotype threat can cause anyone to perform worse when they believe the negative stereotype could be true or worry about confirming a stereotype about a group to which they belong (choice B is correct). Numerous studies have shown that emphasizing the stereotype that males are better at math than females will produce lower math performance for females; this is one of the classic stereotype threat examples (choice A is wrong). Similarly, emphasizing an individual's race will also often lead to poorer academic performance, since the stereotype for most racial and ethnic minorities is that they do not perform as well on standardized tests as white people do; this is also a classic stereotype threat example (choice C is wrong). When females perform math-based assessments in all-female groups, stereotype threat is not activated, because they are less aware of their female-ness while taking the exam, so they do tend to perform much better; this is also a classic stereotype threat example (choice D is wrong).

Light from visible lasers can heat tissue by: A. transferring energy at refractive boundaries. B. being absorbed by chromophores. C. causing friction with the oscillations of light waves. D. stimulating radioactive emission.

B. As stated in the passage, absorption of photons by chromophores transforms their energy into heat and other forms. This is the mechanism of heating by light. Choice A is incorrect because when light refracts, its frequency does not change; thus, neither will its energy, meaning none of it is transferred to the medium. Light does not actually vibrate matter the way sound would, so choice C does not make sense. Visible light does not interact with matter in a way that would have radioactive consequences, so choice D is also incorrect.

Which of the following inhibitors would have no effect on the slope of a Lineweaver-Burk plot? A. Competitive Inhibitor B. Uncompetitive Inhibitor C. Noncompetitive Inhibitor D. Mixed Inhibitor

B. As the slope of the Lineweaver-Burk plot is Km/Vmax, the correct answer is an inhibitor that has the same effect on both Km and Vmax. An uncompetitive inhibitor leads to an equal decrease in the both the Km and Vmax, leading to a series of parallel lines on a Lineweaver-Burk plot (choice B is correct). Competitive, noncompetitive, and mixed inhibitions would all affect the slope of a Lineweaver-Burk plot (choices A, C, and D are wrong).

All of the following are involved in proprioception, EXCEPT: A. muscle spindles. B. baroreceptors. C. Golgi tendon organs. D. joint capsule receptors.

B. Baroreceptors are a type of mechanoroceptor located within the blood vessels sensitive to the stretch of the vessel itself. Baroreceptors detect changes in blood pressure, and are not involved in proprioception (choice B is correct). Muscle spindles are mechanoreceptors sensitive to the stretch of muscles, and are thus involved in movement and proprioception (choice A is wrong). Similarly, the Golgi tendon organs monitor the amount of tension in the tendons and are also involved in movement and proprioception (choice C is wrong). Joint capsule receptors sense pressure, movement, and tension in the joints and are also involved in movement and proprioception (choice D is wrong).

If the shin-byung syndrome is seen to occur across several generations of families who tend to live very close to one another, often in the same house, which of the following would be the best non-genetic reason for the generational repetition? A. Deindividuation B. Observational learning C. Group polarization D. Internal attribution

B. Barring a genetic or proximal environmental cause that would impact the entire family, it is most likely that children observe the behaviors and symptoms of the women afflicted with shin-byung in their family and replicate those behaviors themselves. This reflects the process of observational learning as described by Albert Bandura (choice B is correct). Deindividuation occurs when people lose their sense of self when they are part of a large crowd and are thus made to feel more anonymous. This can lead to irrational and mob-like behaviors (choice A is wrong). Group polarization is the process by which a person's initial attitude on a controversial topic can become more extreme by being part of a group of like-minded people (choice C is wrong). Making an internal attribution means that, when witnessing the behavior of another person, the observer decides that the cause of that person's behavior was internal rather than situational (choice D is wrong).

Certain digestive enzymes secreted by pancreatic acinar cells are synthesized as longer polypeptide chains than they actually possess when carrying out their specific functions. The most reasonable explanation for this observation is that these enzymes: A. are targeted by lysosomal hydrolases for protein degradation. B. are zymogens. C. have not gone through mRNA splicing. D. have been inactivated through cleavage by proteases.

B. Because the digestive enzymes secreted by the pancreatic acinar cells would digest the pancreas itself if secreted in an active form, the enzymes are secreted in an inactive precursor form called a zymogen. Activation of the zymogen usually involves removal of a portion of the protein precursor, therefore the zymogen can be considerably longer than the active form of the enzyme (B is correct). The proteins are to be secreted, not digested by the lysosome (A is wrong), and if they had been cleaved by proteases they would be shorter, not longer (D is wrong). Finally, enzymes do not go through mRNA splicing, mRNA does (C is wrong).

Which of the following is the best technique for separating 2-butanol from propanoic acid? A. 1H NMR spectroscopy B. Fractional distillation C. Infrared spectroscopy D. Recrystallization

B. Both 2-butanol, an alcohol, and propanoic acid, a carboxylic acid, have low molecular weights, so are likely liquids at room temperature. Since carboxylic acids have stronger hydrogen bonding interactions than alcohols, these compounds will have different boiling points. Therefore, fractional distillation would be the best method for separating these molecules. Choices A and C can be eliminated because they are not separation techniques. Crystallization separates solid compounds based on their solubilities and would not be a good choice as both molecules possess similar solubility characteristics.

Which of the following qualities would both a symbolic and a modern racist most likely value? A. Collectivism B. Protestant work ethic C. Egalitarianism D. Liberalism

B. Both a symbolic and a modern racist would most likely value the Protestant work ethic. The ethic represents traditional American values of independence, self-reliance, and hard work (choice B is correct). Modern racists may have an egalitarian self-image, but they disagree with political initiatives such as affirmative action because they perceive these measures as giving an unfair advantage to blacks. Symbolic racists do not present an egalitarian self-image. Instead, they present anti-black sentiment and blame the system for giving black individuals preferential treatment. Egalitarianism would most likely be valued by individuals who exhibit ambivalent racism because they sympathize with the disadvantaged group (choice C is wrong). Collectivism suggests interdependence between groups and between human beings in general and would therefore not be valued by symbolic or modern racists (choice A is wrong). Both symbolic and modern racists are also more likely to value conservative ideals rather than liberal ideals, and to have a conservative outlook (choice D is wrong).

he McGurk Effect is an example of what? A. Weber's Law B. Bottom-up processing C. Working memory D. Visual perception

B. Bottom-up processing is a type of information processing that uses incoming data from the environment to form a perception. When individuals encounter novel stimuli they are forced to engage in bottom-up processing and construct their perception of the stimuli from bits of raw sensory information. With regard to the McGurk Effect, the individual is creating a perception from the raw visual and auditory stimuli that he or she encounters in the environment; this is an example of bottom-up processing (choice B is correct). Weber's law states that the change in a stimulus that will be perceived as just noticeable is a constant ratio of the original stimulus; this law helps explain when we will and will not perceive a difference between two of the same stimuli (e.g., a sound and the same sound at a slightly higher volume). It does not have anything to do with the McGurk Effect (choice A is wrong). Working memory is part of information processing theory. However, it is a component of information processing whereby an individual holds and manipulates information in his or her mind before that information is encoded into long-term memory, which is not integral to the McGurk Effect (choice C is wrong). Finally, visual perception is one way in which human beings receive information from their environment, and is one of many ways that individuals gather data from their surroundings that interact to influence perceptions. Although visual perception is a type of sensory perception that is involved in the McGurk Effect, the effect also relies upon auditory perception to create the illusion (choice D is wrong).

Which of the following equilibria could be present in a buffered solution? A. HCl( aq ) <-> H + ( aq ) + Cl - ( aq ) B. H 2 CO 3 ( aq ) <-> H + ( aq ) + HCO 3 - ( aq ) Correct Answer C. H2O 2 ( aq ) + 2KI( aq ) <-> 2KOH( aq ) + I 2 ( aq ) D. CaCl 2 ( aq ) <-> Ca 2+ ( aq ) + 2 Cl - ( aq )

B. Buffered solutions are commonly made with weak acids and their conjugate bases. The only weak acid given is carbonic acid (choice B). HCl (choice A) is a strong acid and CaCl 2 (choice D) is a salt. Choice C is a redox reaction.

HIV, the virus that causes AIDS, is a retrovirus. Retroviruses have an RNA genome that, after entry into the host cell, is reverse transcribed to DNA, then incorporated into the host genome. RNA copies of the viral genome are produced using the normal host machinery, then packaged into viral capsid proteins for release. Which one of the following is most likely to contribute to the development of drug-resistant HIV? A. Mutation of the virus after insertion into the host-cell genome B. Frequent random errors in transcription by host-cell enyzmes r C. Viral proteins folding differently in the presence of drug than in its absence D. Changes in the tertiary structure of viral RNA by drug

B. Drug resistance develops as mutant versions of the virus are produced and begin infecting new host cells. The change must be able to be passed on to viral progeny. If the viral proteins fold differently in the presence of drug than in its absence, this might account for the mechanism of action of the drug, but does not explain how drug resistance develops, nor could this be a heritable change (choice C is wrong). Changes in the tertiary structure of the viral RNA by the drug is another temporary, non-heritable change (choice D is wrong). Choices A and B are both plausible mechanisms for creating new, heritable versions of the virus. However, mutation of the virus after insertion into the host-cell genome would require that an error in DNA replication be made—a rather unlikely event, given the proofreading ability of DNA polymerase. Viral progeny are produced through transcription (creating a new RNA viral genome off the permanent DNA version inserted into the host-cell genome), and the question states that this occurs using the normal host machinery. Since the normal host RNA polymerases have no proofreading function, it is more likely that errors will occur here, leading to mutant virus (choice B is better than choice A).

An affective disorder involving a pattern of comparatively mild depression that lasts for at least two years is known as: A. cyclothymic disorder. B. dysthymic disorder. C. seasonal affective disorder. D. bipolar disorder.

B. Dysthymic disorder, a less severe pattern of depression, is characterized by the sad mood, lack of interest, and loss of pleasure associated with major depression. These symptoms are milder but longer lasting. To qualify as a dysthymic disorder, the symptoms must last for at least two years in adults and one year in children (choice B is correct). Cyclothymic disorder is a milder form of bipolar disorder and is characterized by an alternating pattern of mood swings (choice A is wrong). Seasonal affective disorder is a calendar-linked pattern of depressive episodes, with depressive symptoms most prominent during months of shorter daylight (choice C is wrong). Bipolar disorder is an affective disorder characterized by alternating extremes of mood, from depression to mania and back again (choice D is wrong).

An individual action potential in a model neuron results in 100 sodium ions crossing the cell membrane. Given that the sodium-potassium pump requires a single molecule of ATP per cycle, how many acetyl-CoA would be required to generate enough energy to extrude the new sodium in the cell via the pump? A. 1 B. 4 C. 5 D. 50

B. Each acetyl-CoA generates 10 ATP equivalents (3 NADH = 7.5 ATP, 1 FADH2 = 1.5 ATP, and 1 GTP = 1 ATP) and the sodium-potassium pump removes three sodium from the cell per cycle at the expense of one ATP. A total of 34 cycles of the pump would be required to remove the 100 sodium from the cell and this would require 34 ATP. To ensure we have sufficient ATP, four acetyl-CoA are needed (choice B is correct).

All of the following statements regarding enzymes are true EXCEPT that they: A. can be stereospecific in substrate recognition. B. alter reaction equilibria. C. are often regulated by feedback inhibition. D. reduce the activation energy of reactions.

B. Enzymes lower the activation energy of reactions to speed them up (choice D is true and eliminated), but they do not alter the potential energy of reactants and products or the reaction equilibrium, only the rate at which equilibrium is reached (choice B is false and the correct response). Enzymes do have great stereospecificity in the reactions they catalyze (choice A is true and eliminated) and are frequently regulated by feedback inhibition by products (choice C is true and eliminated).

The virus herpes simplex I (HSV-I) normally lies dormant in infected individuals. Once activated, the virus enters the productive cycle, leaves the host cell via budding, and causes an outbreak of cold sores in the infected individual. What is the most likely reason that cold sores result? A. After host cell lysis, the host can immediately recognize the virus via antibodies and subsequently removes HSV-I infection from the body; cold sores are just a side effect of this action. B. After budding, the virus envelopes itself in the cell membrane of the host; this decreases host recognition of HSV-I, increases the length of time the virus can spread, and leads to a breakout of cold sores. C. After host cell lysis, the host cannot immediately recognize the virus itself and HSV-I goes on to infect other cells, causing a breakout of herpes cold sores around the mouth. D. After the virus is activated, it causes the host cell to release large amounts of hormones; this sudden regional hormone imbalance causes cold sores.

B. HSV-I is able to lie dormant in host cells, so it must undergo the lysogenic cycle. The question stem also says that the virus enters the productive cycle when it is activated, meaning the new viral particles bud out of the host cell instead of lysing the host cell (choice A and C discuss host cell lysis and can be eliminated). When the virus buds out of the host cell, it acquires an envelope that is derived from the host cell plasma membrane. As a consequence, the host immune system can recognize the virus as "self" instead of "foreign," and therefore may not immediately attack it (choice B is correct). While a sudden hormone imbalance might trigger activation of HSV-I, cold sores are not caused by sudden regional hormone imbalance as a direct result of virus activation (choice D is wrong).

A child who learns to avoid approaching her parent for help and comfort because she is always met with aggression by the parent develops an anxious-avoidant insecure attachment style. The child thus associates the parent with fear and begins to actively avoid interactions with the parent in order to avoid this emotional experience. This type of learning can also be explained by which operant conditioning principle? A. Modeling B. Avoidance learning C. Vicarious learning D. Role-playing

B. In avoidance learning, an organism's behavior is reinforced by the termination or prevention of an aversive stimulus. In other words, the individual is motivated to escape fear-producing environmental stimuli or conditioned stimuli. In this instance, because the child is always met with anger when she encounters the parent, the parent becomes a conditioned stimulus that produces fear. The child thus begins to actively avoid interactions with the parent in an effort to avoid experiencing fear (choice B is correct). Observational learning, also known as modeling or vicarious learning, is part of Albert Bandura's social learning theory. Bandura believed that learning could occur by watching others. However, not all observed behaviors are effectively learned. In order for the modeling process to work, according to Bandura, the following factors have to be present: attention, retention, reproduction, and motivation. The child in question is avoiding the parent and does not appear to be motivated to reproduce the aggressive behavior that she is observing (choices A and C are wrong). Role-playing is not an operant conditioning principle. Rather, it is an instance or situation in which one deliberately acts out or assumes a particular character or role. In psychology, it is a therapeutic technique designed to reduce conflict in social situations; participants act out particular behavioral roles in order to expand their awareness of differing points of view. Role-playing has also been the focus of many social psychology studies (choice D is wrong).

In transposition of the great arteries, a congenital birth defect, the aorta of the newborn is connected to the right ventricle and the pulmonary artery is attached to the left ventricle. This would result after birth in circulation of: A. oxygenated blood through the systemic vasculature and deoxygenated blood through the pulmonary vasculature. B. deoxygenated blood through the systemic vasculature and oxygenated blood through the pulmonary vasculature. C. oxygenated blood through both the systemic and pulmonary vasculature. D. deoxygenated blood through both the systemic and pulmonary vasculature.

B. In the normal circulation, the right ventricle pumps blood through the pulmonary artery to the lungs. Blood returns from the lungs to the left atrium and is pumped by the left ventricle through the aorta to the rest of the body. If blood from the left ventricle passes to the pulmonary artery instead of the aorta and is then returned to the left atrium, oxygenated blood will loop around through the pulmonary system without passing to the rest of the body. At the same time, the deoxygenated blood from the systemic circulation will pass to the right atrium, then back out to the systemic circulation through the aorta rather than the pulmonary artery, causing deoxygenated blood to loop through this system (choice B is correct and choice A is wrong). Choices C and D are wrong since they have all blood either oxygenated or deoxygenated.

In people suffering from color blindness, which of the following types of cells fails to function normally? A. Rod cells B. Cone cells C. Neurons of the optic nerve D. Corneal cells

B. Rods and cones are the photoreceptor cells of the retina. Rods are the ones more sensitive to light, making them important for night vision. Cones are the cells responsible for detecting color and are the cells responsible for the defect involved in color blindness (choice A is wrong, and choice B is correct). The optic nerve is responsible for all vision, making it unlikely that a defect in the optic nerve would affect only color vision and not signals from rod cells (choice C is wrong). Corneal cells are part of the mechanical structure of the eye but are not photoreceptors and are not likely to specifically affect color vision (choice D is wrong).

A person is presented with four unlabeled boxes that are 8, 9, 11, and 12 pounds each and one 10-pound box that is labeled. He first picks up the 10-pound box, followed by the 9-pound box, and says that they are the same weight. He then picks up the 10-pound box, followed by the 11-pound box, and says that they are the same weight as well. However, when he compares the 8 and 10-pound boxes, he notices that the 8-pound box is lighter, and when he compares the 12 and 10-pound boxes, he notices that the 12-pound box is heavier. Which of the following is true in this scenario? A. The absolute threshold is10 pounds B. The difference threshold is 2 pounds C. The difference threshold is 4 pounds D. The just noticeable difference is 4 pounds

B. In this scenario, the 10-pound box is a standard stimulus and the others are comparison stimuli. The two-pound difference between 8 and 10 and between 10 and 12 that is required for the person to tell that the boxes are different weights is the difference threshold, or the amount of energy that needs to be added to, or subtracted from, the standard stimulus before there appears to be a difference (choice B is correct; choice C is wrong). An absolute threshold is the amount of stimulus energy needed for a person to perceive it at all, which is not discussed in this scenario (choice A is wrong). The just noticeable difference (JND) is the same value as the difference threshold; in this case, two pounds would equal one JND, four pounds would equal two JNDs, etc. (choice D is wrong).

When a corporation chooses the location of new corporate facilities, which will provide employment opportunities for the surrounding community, based on the racial composition of the local neighborhoods and stereotypes about various minority groups, this corporation is demonstrating: A. individual discrimination. B. institutional discrimination. C. structural discrimination. D. reverse discrimination.

B. Institutional discrimination is the unjust treatment of a particular group by societal institutions; the passage mentions that some corporations intentionally factor racial composition and stereotypes into work decisions (choice B is correct). Unlike institutional discrimination, individual discrimination is the unjust treatment of a particular group by individuals (choice A is wrong). Structural discrimination is similar to institutional discrimination; however, structural discrimination occurs when individuals who run the institutions in question do not intend to treat any group differently or unjustly, but inadvertently do so through their established practices (choice C is wrong). Reverse discrimination is the unjust treatment of the dominant group; in the scenario presented, it is the minority group that is being negatively affected in terms of employment opportunities (choice D is wrong).

Increased intermolecular attractions cause the ratio PV/(RT) to decline because individual molecules: A. eventually combine with other molecules, decreasing the number of particles in the container. B. lose kinetic energy to potential energy and strike the side of the container with less force. Correct Answer C. increase in speed due to electron repulsion and strike the side of the container with increased force. D. transfer electrons during collisions with other molecules in the container.

B. Intermolecular attractions increase the potential energy between molecules, decreasing their kinetic energy. Note that the molecules do not undergo any reactions, eliminating choices A and D. Choice C is incorrect because intermolecular attractions do not result in electron repulsion.

Which of the following is correct regarding the indicated biochemical pathway and its products? The Krebs cycle produces NADH and FADH2 following oxidation of acetyl-CoA carbons. Beta-oxidation produces NADH and acetyl-CoA following oxidation of acyl-CoA. The electron transport chain produces NADH and ATP following oxidation of electron carriers. A. I only B. I and II only C. II and III only D. I, II, and I

B. Item I is true: The Krebs Cycle produces NADH and FADH2, as well as CO2, which result from oxidation of carbons present on acetyl-CoA (choice C can be eliminated). Item II is true: beta-oxidation involves oxidation of acyl-CoA molecules and yields NADH, FADH2, as well as acetyl-CoA as products. Note that choice A does not include Item II (choice A can be eliminated). Item III is false: the electron transport chain does produce ATP as the result of oxidation of electron carriers, but oxidation of those carriers would yield NAD+, not NADH (choice D can be eliminated and choice B is correct).

Viral infection would lead to an increase in which of the following cellular pathways? Aerobic respiration Pentose phosphate pathway cAMP mediated protein kinase activation A. I only B. I and II only C. II and III only D. I, II, and III

B. Item I is true: viral infected cells are undergoing a fair amount of cellular activity, including DNA replication, RNA transcription, and protein synthesis, all of which contribute to an increased need for ATP, and thus oxidative respiration (choice C can be eliminated). Item II is true: the pentose phosphate pathway produces essential precursors for nucleotide synthesis, necessary for both DNA replication and RNA transcription. It's likely that the activity of this pathway would be increased during viral infection (choice A can be eliminated). Item III is false: cAMP mediated protein kinase activity is increased when G protein coupled receptors are bound and activated; there is no reason to assume this in viral infection (choice D can be eliminated and choice B is correct).

Muscle fibers are composed of small contractile units called sarcomeres. During contraction, which of the following occurs within a sarcomere? Myosin filaments shorten. Actin filaments shorten. Overlap between actin and myosin filaments increases. A. I only B. III only C. II and III only D. I, II, and III

B. Items I and II are false: During contraction, neither myosin nor actin filaments get shorter. The overlap between them increases to make the sarcomere shorter (eliminate choices A, C and D). Item III is true: The overlap between the fibers increases as part of contraction.

When the oxidizer and the fuel react together to produce an explosion, the resulting sound waves are: A. longitudinal, causing air molecules to move perpendicular to the direction of wave propagation. B. longitudinal, causing air molecules to move parallel to the direction of wave propagation. C. transverse, causing air molecules to move perpendicular to the direction of wave propagation. D. transverse, causing air molecules to move parallel to the direction of wave propagation.

B. Sound waves are longitudinal, which means that the air molecules oscillate in the direction parallel to the direction of wave propagation.

Which of the following lipids is a key component of myelin sheaths? A. Terpenes B. Sphingolipids C. Prostaglandins D. Tocopherols

B. Sphingomyelin, a type of sphingolipid, is an important lipid component in the myelin sheath of nerve cell axons (choice B is correct). The other compounds do not play an important role in myelin (choices A, C, and D are incorrect).

Which of the following does NOT correctly describe the control of gene expression in eukaryotes? A. Methylation of DNA prevents the binding of transcription factors and hinders gene expression. B. The promoter region is the start point of transcription and contains a binding site for RNA polymerase that can be modified by repressors and enhancers. C. One method of inactivating DNA transcription in eukaryotes involves methylation of histones and increased heterochromatin formation. D. RNA interference (RNAi) is a posttranscriptional method of regulating gene expression.

B. Methylation of DNA (called imprinting) and methylation of histones with subsequent heterochromatin formation can both silence gene expression (choices A and C correctly describe methods of controlling gene expression and can be eliminated). RNAi is mediated by siRNA (small interfering RNA) or miRNA (micro RNA). These small non-coding RNAs bind to complementary sequences on mRNA and the resulting double stranded RNA is degraded. The reduction in mRNA leads to a reduction in the gene product (choice D correctly describes a method of controlling gene expression and can be eliminated). However, the promoter is just the binding site for RNA pol; the start point of transcription is located downstream from the promoter, and typically the regulatory regions for repressor/enhancer binding are between them (choice B does NOT correctly describe the regulation of gene expression and is the correct answer choice).

With which of the following processes is a double-stranded DNA break NOT typically associated? A. Recombination B. Mitosis C. Balanced Robertsonian translocation D. Unbalanced Robertsonian translocation

B. Mitosis does not typically involve double-stranded DNA breaks, although they can occur by chance (choice B is the correct answer). Recombination (crossing over) in meiosis begins with a double-stranded DNA break, followed by invasion of the opposite strand (choice A involves a double-stranded break and can be eliminated). Both balanced and unbalanced Robertsonian translocations must involve double-stranded DNA breaks in order for two chromosomes to exchange long arms (choices C and D can be eliminated). The only difference between balanced and unbalanced translocations is that in an unbalanced translocation some genetic information (DNA) is gained or lost.

The acidity of hydrogen halides increases as one moves down the periodic table. Which of the following properties of halides is most useful in explaining this trend? A. Boiling point B. Ionic radius C. Density D. Atomic weight

B. Moving down the periodic table, the ionic radii of the atoms increase. As the ionic radius increases, the stability of the corresponding anion also increases (since the negative charge is distributed over a larger volume). Therefore, since larger halides are more stable as anions, larger hydrogen halides are more likely to undergo deprotonation, and are stronger acids.

The following characteristics are shared between skeletal and smooth muscle with the exception of: A. Upstroke in action potential is secondary to inward Na+ current B. Potential exhibits a plateau C. Action potential opens voltage gated Ca2+ channels D. Ca2+ is the primary substrate for triggering contraction

B. Neither skeletal nor smooth muscle contraction is initiated by an action potential with a plateau; this feature is only characteristic of cardiac action potentials (answer choice B does not indicate a similarity and is the correct answer). The upstroke in the action potential for both skeletal and smooth muscle is triggered by an inward Na+ current (answer choice A states a similarity and is not the correct answer). Once received by either skeletal or smooth muscle, the action potential does, indeed, cause voltage-gated Ca2+ channels to open; in skeletal muscle, these channels are located on the sarcoplasmic reticulum while, in smooth muscle, they are located in the cell membrane (answer choice C is wrong). Indeed, Ca2+ is the molecular substrate for triggering contraction in both skeletal and smooth muscle; binding with troponin triggers skeletal muscle contraction and calmodulin binding activates myosin light-chain kinase in smooth muscle (answer choice D is wrong).

A semipermeable membrane separates an aqueous solution of 0.003 MNaCl from an aqueous solution of 0.0025 MBaF2. With this apparatus, one would observe that water crosses the membrane: A. from the BaF 2 solution to dilute the NaCl solution. B. from the NaCl solution to dilute the BaF 2 solution. C. from the BaF 2 solution to concentrate the NaCl solution. D. from the NaCl solution to concentrate the BaF 2 solution.

B. Osmotic pressure is a colligative property, which means it depends only on the concentration of solute particles, and not their identity. Therefore, a 0.003 M solution of NaCl will have an effective molality of 0.006 M (since NaCl is composed of two ions) and a 0.0025 M solution of BaF2 will have an effective molality of 0.0075 M (since BaF2 is composed of three ions). Osmotic pressure will drive movement of water from the lower concentration solution (NaCl) to the higher concentration solution (BaF2). Note that this movement of water acts to dilute the BaF2 solution.

Perhaps an individual participating in this study struggled with insomnia one night and now believes the condition will persist into the future. This is an example of: A. self-actualization. B. overgeneralization. C. the Pygmalion effect. D. a self-fulfilling prophecy.

B. Overgeneralization occurs when an individual comes to a conclusion based on one episode or bit of evidence. In this scenario, the subject has recently experienced a single episode of insomnia and consequently believes he or she is prone to suffering from it in the future (choice B is correct). Self-actualization involves the maintaining of a positive worldview that enables the individual to reach his or her fullest potential (choice A is wrong). A self-fulfilling prophecy occurs when a prediction comes true because it is believed to be true and thus inspires behavior, attitudes, etc. that bring about its own realization. In this case it is unknown whether the subject will in fact continue to experience insomnia (choice C is wrong).The Pygmalion effect is closely related to the self-fulfilling prophecy; the terms are often considered synonymous (choice D can be eliminated).

In order to understand deception in academia, researchers conducted an experiment in which student participants were asked to request an extension on an exam from their respective professors. The students were assigned to one of three conditions describing the reason for needing the extension: (1) the student was not feeling well after drinking too much, (2) the student was recently at the hospital with the flu, or (3) the student was anxious about balancing too much work. The student was able to provide either an honest or deceptive explanation in the attempt to be granted the extension. Students assigned to which condition are most likely to provide an honest response? A. Condition one B. Condition two C. Condition three D. It cannot be determined.

B. Paragraph 2 of the passage offers possible explanations for deception. In situations wherein an individual does not conform to the social norms, the truth might be seen as unacceptable and lies are told as a form of self-protection. Therefore, in the research described in the question stem, the students with legitimate excuses (those that are acceptable according to social standards) are expected to present the truth, while the students with illegitimate excuses (those that are unacceptable according to social standards) are expected to conceal the truth. The most acceptable excuse is that provided to students assigned to condition two ("the student was recently at the hospital with the flu"); thus, these participants are most likely to provide their professors with an honest reason for the request (choice B is correct and choice D is wrong). The least acceptable excuse is that provided to students assigned to condition one ("the student was not feeling well after drinking too much"); thus, these participants are least likely to provide their professors with an honest reason for the request (choice A is wrong). Excessive drinking is seen as a form of social deviance that is not to be condoned, and there is a social stigma attached to it. Finally, the excuse that is provided to students assigned to condition three ("the student was anxious about balancing too much work") is moderately unacceptable. Accordingly, these participants are not those most likely to provide their professors with an honest reason for the request (choice C is wrong). Similar to excessive drinking, there is often a social stigma attached to expressions of emotional struggle, especially in formal settings (e.g., an academic setting).

What substance might be expected to be in mosquito saliva that could inhibit the process of hemostasis? A. A vasoconstrictor B. An inhibitor of platelet aggregation C. A stimulator of coagulation D. A stimulator of T cells

B. Platelets are an essential part of the hemostatic process; they form large clumps (aggregation) as part of clot formation. Thus an inhibitor of platelet aggregation could inhibit hemostasis (choice B is correct). Vasoconstrictors constrict blood vessels to slow bleeding and can contribute to hemostasis (choice A is wrong). Coagulation is the process by which clots form to stop bleeding; a stimulator of coagulation would not inhibit hemostasis (choice C is wrong). T cells do not play a role in coagulation (choice D is wrong).

The psychiatric term for a person's "loss of contact with reality" is: A. splitting. B. psychosis. C. defense mechanism. D. paranoia

B. Psychosis is properly defined as a "loss of contact with reality" (choice B is correct). Splitting is a term used to describe "all or nothing" thinking, in which a person vacillates between thinking of the positive and negative aspects of situations or people, but cannot bring them into a nuanced whole. Psychiatrists consider this a defense mechanism, and it is a trait associated with borderline personality disorder (choice A is wrong). A defense mechanism is an unconscious coping strategy that a person uses to deal with a stressful situation (choice C is wrong). Paranoia is a condition characterized by anxiety, suspicions of conspiracy, perceived threats, and distrust of others (choice D is wrong).

Which of the following would be true about cis-oleic acid, a monounsaturated fatty acid with the formula CH3(CH2)7(CH)2(CH2)7COOH? A. It will generate approximately 119 ATP after 9 rounds of β-oxidation, followed by the Krebs cycle and the electron transport chain. B. It will generate approximately 119 ATP after 8 rounds of β-oxidation, followed by the Krebs cycle and the electron transport chain. C. It will generate 90 ATP after 9 rounds of β-oxidation, followed by the Krebs cycle and the electron transport chain. D. It will generate 90 ATP after 8 rounds of β-oxidation, followed by the Krebs cycle and the electron transport chain.

B. Recognize the ability to treat this question as a 2x2 elimination. cis-Oleic acid has 18 carbons and thus will undergo 8 rounds of β-oxidation (eliminate choices A and C). This will generate 9 molecules of acetyl-CoA, 8 molecules of NADH and 7 molecules of FADH2 since it is a monounsaturated fatty acid. Each of the 9 acetyl-CoAs will go through the Krebs cycle and this will generate 27 NADH (which will give 67.5 ATP), 9 FADH2 (which will give 13.5 ATP) and 9 GTPs (9 ATP equivalents). This means the acetyl-CoAs alone generate 90 ATP equivalents. Since the NADH and FADH2 made in β-oxidation will generate even more ATP, the total will exceed 90 (eliminate choice D and choice B is correct). The 8 molecules of NADH made in β-oxidation will lead to 20 ATP and the 7 FADH2 will give 10.5. Also remember that fatty acid activation (which must occur before β-oxidation) costs the cell two high energy bonds, or ATP equivalents. This means the electron carriers made in β-oxidation will give a net yield of 28.5 ATP. Overall then, cis-oleic acid will generate 118.5 ATP molecules.

In cellular respiration, which step causes the difference in ATP yield occur between prokaryotes and eukaryotes and by how many ATP? A. The difference occurs in glycolysis and is 2 ATP. B. The difference occurs in the electron transport chain and is 2 ATP. C. The difference occurs in glycolysis and is 4 ATP. D. The difference occurs in the electron transport chain and is 4 ATP.

B. Recognize the opportunity to handle this question as a 2x2 elimination. The difference in ATP yield between prokaryotes and eukaryotes is 2, with prokaryotes producing 32 ATP and eukaryotes producing 30 ATP (choices C and D are wrong). In prokaryotes, the 2 NADH produced in glycolysis have direct access to the cell membrane where the electron transport chain occurs. However in eukaryotes, the electrons from glycolytic NADH must be transported into the mitochondria before they can enter the electron transport chain; furthermore they do not begin their interaction with the ETC until its second protein (coenzyme Q, choice A is wrong and choice B is correct). Glycolysis has the same output in both prokaryotes and eukaryotes.

Retroviruses, which are a subclass of RNA viruses, are unique in that they contain: A. DNA-dependent DNA polymerase. B. RNA-dependent DNA polymerase. C. DNA-dependent RNA polymerase. D. RNA-dependent RNA replicase.

B. Retroviruses have RNA genomes but undergo the lysogenic cycle in hosts with double-stranded DNA genomes. Therefore, the virus must be able to reverse transcribe its genome into DNA so that it can successfully integrate into the host-cell genome. Enzymes that are able to create a strand of DNA by reading a strand of RNA are called RNA-dependent DNA polymerases (B is correct). DNA-dependent DNA polymerases make a strand of DNA by reading a strand of DNA and are what the host cell normally uses for replicating its own genome (A is wrong). DNA-dependent RNA polymerases can make a strand of RNA by reading a strand of DNA and are what the host cell uses to transcribe its genome (C is wrong), and there is no such thing as an RNA-dependent RNA replicase (D is wrong).

According to self-determination theory, which proposes that people will seek autonomy, competence, and relatedness throughout their lifespans, the oldest participants in the study would most likely: A. determine on their own that they are unfit to drive, regardless of their objective driving performances. B. emphasize that any decisions to limit their respective driving privileges are self-endorsed. C. strive to achieve a sense of acceptance regarding their individual life histories. D. enhance their driving skills at the expense of social relationships.

B. Self-determination theory maintains that, throughout the lifespan and especially in older age, people seek autonomy, competence, and relatedness. Autonomy refers to viewing one's circumstances as self-endorsed, while competence refers to the expression of one's talents. Finally, relatedness refers to a sense of being cared about by other people. Therefore, according to this theory, one of the oldest participants in this study would emphasize that a decision to limit his or her driving is self-endorsed (choice B is correct). A determination of being unfit irrespective of driving skill is not implied by self-determination theory (choice A is wrong). Enhancing one's skills at the expense of social relationships contradicts the theory's principle of relatedness (choice D is wrong). Finally, achieving a sense of acceptance of one's personal life history is associated with Erikson's developmental conflict of integrity versus despair, not with self-determination theory (choice C is wrong).

"M" likes and respects her therapist, who has stated repeatedly that M suffers from dissociative identity disorder (DID) and likely has several alternate personalities, although M remains skeptical. When M concedes that a particular memory lapse "could possibly have been due to the appearance of an alternate personality," her therapist praises her for being brave enough to confront the painful reality of her condition. When asked to imagine what one of her alternates might be like, M describes a six-year-old girl, and her therapist praises her as before. During a subsequent session M allows her therapist to "speak with" this girl, after which the therapist praises her profusely. M now believes that she suffers from DID. The process by which M came to accept the DID diagnosis might be explained as: A. social learning therapy. B. shaping. C. negative reinforcement. D. aversive conditioning.

B. Shaping is an operant conditioning technique that involves producing a desired behavior (in this case acceptance of the DID diagnosis) by reinforcing successive approximations of that behavior (choice B is correct). Social learning therapy involves an attempt to change an individual's behavior by having him or her observe a model being reinforced for exhibiting desirable responses (choice A is wrong). Negative reinforcement is an operant conditioning technique in which the frequency of a desired behavior is increased by following that behavior with the removal of an unpleasant or undesirable stimulus. Praise from her therapist would be a desirable stimulus for M (choice C is wrong). Aversive conditioning is a behavior modification technique that attempts to eliminate an unwanted behavior by pairing it with an unpleasant stimulus (e.g., administering a mild electric shock each time the subject takes a sip of an alcoholic beverage; choice D is wrong).

Which of the following is most correct regarding eukaryotic chromosomal organization? A. Euchromatin is highly condensed, transcriptionally active DNA found in the eukaryotic nucleus. B. The histones found in eukaryotic chromosomes contain a relatively large amount of lysine and arginine amino acids. Correct Answer C. Euchromatin has equal transcriptional activity when compared to heterochromatin, however heterochromatin displays greater steric hindrance to DNA transcription proteins. D. Eukaryotic chromosomes can be found in the cytosol during transcription.

B. Since DNA is largely negatively charged due to the many phosphate molecules in its nucleic acid backbone, the histones, which allow DNA to be tightly condensed, would contain many positively charged amino acids, such as arginine and lysine (choice B is correct). Euchromatin is loosely condensed, relative to heterochromatin, to allow a high transcriptional activity (choice A is wrong). It has greater transcriptional activity compared to heterochromatin (choice C is wrong). Both transcription, as well as eukaryotic chromosomes, are found inside the nucleus (choice D is wrong).

Which of the following is the LEAST characteristic of the Krebs cycle? A. Oxaloacetate is combined with acetyl-coA to produce citrate. B. Most of the energy is produced through substrate-level phosphorylation. C. Most of the energy is produced indirectly through high-energy electron carriers. D. 6 NADH, 2 FADH2, and 2 GTP are produced per glucose.

B. Since the question is looking for the item that is LEAST characteristic of Krebs cycle, eliminate choices that DO characterize it. Choice A is true and can be eliminated: oxaloacetate is combined with acetyl-CoA to form citrate (thus why the cycle is sometimes referred to as the citric acid cycle). Choice C is also true and can be eliminated: most of the energy is produced from the production of high-energy electron carriers (NADH and FADH2). Approximately 18 of the 20 ATP that are associated with Krebs are from these high-energy electron carriers (after they are oxidized in the electron transport chain). Choice D is also true and can be eliminated: six NADH, two FADH2, and two GTP are produced in the Krebs cycle per glucose. Choice B is the only false statement (which is the correct answer for this question). Only two of the 20 ATP made from the Krebs cycle are from substrate-level phosphorylation.

Which of the following is LEAST likely to lead to social loafing? A. Lack of individual evaluation B. Working in cohesive groups C. Incommensurate compensation D. Submaximal goal setting

B. Social loafing is a phenomenon whereby individuals exert less effort on a task when working in a group than they would if working alone. However, if the group is a cohesive unit, individuals are more likely to be concerned about the outcomes. This is especially true of in-groups, in which the members identify with one another and are more likely to make an effort to ensure that they meet the expectations of their fellow members (choice B is correct). If group members are not being evaluated on an individual basis, they tend to feel less accountable for their work. For example, if a group of five medical students were to receive a single grade for a project, each would likely feel less compelled to contribute than he or she would if group members were receiving individual grades (choice A is wrong). If group members are not receiving the same level of compensation, their efforts will likely reflect such disparity. For instance, if one team member is receiving $5 for her efforts, she will probably minimize those efforts if she knows other participants are receiving higher payouts (choice C is wrong). Submaximal goal setting occurs when group members decrease their efforts because they do not believe that they have to work as hard when others are working with them towards the same goal (choice D is wrong).

After asking the opinions of several medical students about raising the voting age, a researcher then asked those participants who were against such a proposal to write a brief essay in support of raising the voting age. As an incentive, he paid half of the medical students $1 and the other half $50. Next, he reassessed their opinions on the topic to see if writing the essay had made an impact. Which group was more likely to change opinions? A. Students paid $50 were more likely to change their opinions because of the conditioning. B. Students paid $1 were more likely to change their opinions because of cognitive dissonance. C. The two groups of students were equally likely to change opinions as the result of exposure to new information. D. No students would change their opinions because this was not a personal topic.

B. Students who are paid to advocate a given position are more likely to support that position when asked about their opinions if the incentive received was minimal, as they are more likely to attribute their opinion change to internal reasons rather than to an external circumstance (i.e., the compensation). Thus, cognitive dissonance causes a shift in opinion (choice B is correct). With respect to answer choice A, when students are given a substantial incentive (in this case $50) to advocate a conflicting position they are able to attribute their advocacy to the large reward given, rather than to internal factors; the pressure of cognitive dissonance is thus removed. Moreover, neither classical nor operant conditioning principles apply to this scenario. Classical conditioning requires the pairing of a neutral stimulus with an unconditioned stimulus until the two stimuli become associated, which is not pertinent to this situation. Operant conditioning involves increasing or decreasing the frequency of a certain behavior by repeatedly rewarding or punishing it, respectively, which did not occur in this case (choice A is wrong). There would be a greater cognitive dissonance effect on students without the external motivation to modify their positions (the group paid $1) than on those with an external explanation for the change (those paid $50). Moreoever, no new information was provided (choice C is wrong). The personal nature of the topic would not be relevant; so long as the students did not have an external factor to which to attribute their advocacy, they would experience cognitive dissonance and would be less supportive of their previous positions (choice D is wrong).

Which of the following best describes how the brain determines the location of a sound? A. The angle of the sound wave, with respect to the eardrum, allows the brain to determine the location of the sound. B. Sound waves reach the ear closest to the source of the sound first, and this difference in timing allows the brain to determine the location of the sound. C. The angle of the sound wave, with respect to the auditory hair cells, allows the brain to determine the location of the sound. D. Sound waves closer to the source have a higher frequency, and this difference in frequency allows the brain to determine the location of the sound.

B. The angle of the sound wave with respect to the ear is not pertinent to determining the location of the sound (choices A and C can be eliminated). Sound waves reach the ear closest to the source of the sound first, and the further ear second. This time gap allows the brain to determine the location of the sound (choice B is correct). Choice D is incorrect; the frequency of the sound is not relevant to determining the location, nor is there a difference from one ear to the next.

DNA from resistant bacteria in Dish 6 is extracted and placed on agar with phage-sensitive E. coli. After incubation it is determined that these E. coli are now insensitive to phage T1 infection. The most likely mechanism for their acquisition of resistance is: A. transduction. B. transformation. C. sexual reproduction. D. conjugation.

B. The definition of transformation is the process in which naked DNA, not a virus, is taken into a cell and changes the genetic characteristics of that cell. That is the case in this experiment, with extracted DNA making cells resistant to the virus (choice B is correct). Transduction is mediated by a virus, conjugation involves direct transfer of DNA between bacteria, and sexual reproduction does not apply to bacteria (choices A, C, and D are wrong).

Sodium laurel sulfate, a common ingredient in hand soap, functions due to the combination of its hydrophilic head (pKa = 1.9) and hydrophobic tail. This amphipathic nature allows for the removal of hydrophobic substances with water. Which of the following would most likely increase the effectiveness of sodium laurel sulfate? A. Benzene B. Dilute aqueous sodium bicarbonate C. Aqueous phosphoric acid (pH = 1.9) D. Lemon juice

B. The head of sodium laurel sulfate (SLS) is a weak acid. It is most hydrophilic when deprotonated and charged. This head can be protonated in lower pH solutions and become less effective. Therefore, addition of a hydrophilic weak base like sodium bicarbonate (NaHCO3) would favor the deprotonated, active form of SLS. Choices C and D are both acidic and eliminated. Benzene is a hydrophobic compound with essentially non-acidic protons (pKa of about 43), eliminating choice A.

Given that the McGurk Effect relies upon visual and auditory stimuli, which of the following brain structures is NOT involved in the processing of the information that produces the McGurk Effect? A. Occipital lobe B. Postcentral gyrus C. Lateral geniculate nucleus in the thalamus D. Cochlear nucleus

B. The postcentral gyrus is a prominent structure in the parietal lobe. The primary function of the postcentral gyrus is the processing of the sense of touch, not the senses of sound or sight, which are involved in the McGurk Effect (choice B is not involved in the processing of information that produces the McGurk Effect and is therefore correct). The occipital lobe is the visual processing center of the brain and contains the visual cortex, which would be involved in the visual processing of the McGurk Effect (choice A is wrong). The lateral geniculate nucleus in the thalamus is involved with the visual processing pathway of the brain. It is the primary relay center for visual information received from the retina of the eye, and therefore would be involved in the visual processing of the McGurk Effect (choice C is wrong). The cochlear nuclei are collections of neurons that receive input from the sound received by the cochlear nerve, and then output that information to the auditory cortex of the brainstem. As the cochlear nuclei are involved in auditory processing, they would be involved in the processing of the McGurk Effect (choice D is wrong).

E. coli is a generally harmless bacterium, often used for research purposes. Some strains of E. coli live mutualistically inside human digestive tracts. Sometimes these bacteria can cause infection outside the iinfection outside the intestinal tract, most notably in the urinary tract, the biliary tract, and the nervous system. In which of the following structures is E. coli most likely to cause infection? A. Trachea B. Bladder C. Small intestine D. Large intestine

B. The question states that E. coli can cause disease outside of the intestinal tract (choices C and D are wrong), in the urinary tract, the biliary tract, and the nervous system. The bladder is the best answer since this is part of the urinary tract (choice B is correct), which was mentioned as a potential site of infection while the respiratory system, and thus the trachea, was not (eliminate choice A).

The volume of a real gas under any set of conditions is smaller than the volume of an ideal gas under the same conditions. This is true because: A. the intermolecular forces of real gases are stronger than those of ideal gases. B. the volume of real gas particles is significant and reduces the free space in the vessel. C. the electron clouds of ideal gas particles repel each other more strongly than those of real gases. D. the intermolecular forces of ideal gases are stronger than those of real gases.

B. The volume of a gas is defined as the free space within the vessel. Since ideal gas particles have insignificant volume, the free space is equal to the container volume. The free space for real gases is reduced by the volume of the real gas particles and therefore is always smaller than an ideal gas under the same conditions. This makes choice B the correct answer. Both choices A and D refer to intermolecular forces, which affect the pressure deviation of real gases, not volume. While ideal gas particles have little to no interactions, they will experience attractive intermolecular forces rather than repulsive ones (eliminate choice C).

An ideal gas contained in a piston begins with pressure P0, volume V0, and temperature T0. It undergoes isothermal expansion to volume 2V0. It then undergoes isobaric compression back to volume V0. Finally, its pressure is increased by an isochoric process until it returns to its original state. Which of the following best approximates the work done by the gas on the environment during this sequence? A. W = 0. B. 0 < W < P0V0 / 4 C. P0V0 / 4 < W < P0V0 / 2 D. W > P0V0 / 2

B. The work done during a thermodynamic process is the area enclosed by the set of process curves on the PV diagram. In this case, three vertex points are given or implied: (P0, V0), (P0 / 2, 2V0), and (P0 / 2, V0), where the value of the pressure after the isothermal (constant temperature) expansion is implied by the fact that PV is constant (Boyle's law): if volume doubles, pressure must halve. To estimate the area contained by the three curves, plot the three points on a PV graph and connect them to form a triangle. Note that this will overestimate the area, because the isotherm is a hyperbola, not a straight line (see the PV graph below). The formula for the area of a triangle is one half base times height, yielding 1/2 × P0 / 2 × V0 = P0V0 / 4 (which, again, is an overestimation).

There are five recognized tastes: sweet, salty, bitter, sour, and umami. Recent research suggests there is also a sixth, oleogustus, the unique taste of fat. The lead researcher on the study said that since there aren't any words that exist for this taste, they were forced to make it up, and it was very difficult to figure out if people really view this as unique sensation. This best demonstrates: A. gestalt principles. B. the Sapir-Whorf hypothesis. C. the major limitations of sensory processing. D. monocular cues.

B. This example suggests that it was difficult to describe and define things for which we don't have language; this is an example of the Sapir-Whorf hypothesis, which suggests that the structure of a language determines/influences the thought and behavior characteristic of the culture in which it is spoken. In other words, without an actual word to define the fatty taste, it was difficult to determine that it was in fact a unique taste (choice B is correct). Gestalt psychology emphasizes the mental processes that perceive the "whole" of objects or situations. This school of psychology has been particularly influential in describing tendencies of human visual perception. Gestalt principles do not describe how a lack of language con influence perception (choice A is wrong). This example describes a major limitation of language to define a sensation, not a limitation of sensory processing itself (choice C is wrong). Monocular cues, such as relative size and texture gradient (among others) are cues that can be perceived with only one eye that help us judge distance (depth perception). They have nothing to do with taste (choice D is wrong).

Can glucogenic amino acids be converted into glucose? A. Yes: pyruvate and oxaloacetate can be converted directly into glyceraldehyde-3-P, which is a major intermediate in both gluconeogenesis and glycolysis. B. Yes: pyruvate and Krebs cycle intermediates can be converted into oxaloacetate, then phosphoenolpyruvate, which can enter gluconeogenesis. Correct Answer C. No: pyruvate and Krebs cycle intermediates are formed as part of glucose breakdown and this process is important to generate ATP for the cell. D. No: glucose is obtained from the diet and stored in the liver; it cannot be made as a new molecule because cellular respiration has several steps with a -ΔG.

B. This is a typical two by two question. The passage says that glucogenic amino acids are broken down into citric acid cycle intermediates or pyruvate. The Krebs cycle regenerates oxaloacetate (OAA), and in the first step of gluconeogenesis, pyruvate is also converted into OAA (by the enzyme pyruvate carboxylase). OAA is converted into phosphoenolpyruvate (PEP) by the enzyme phosphoenolpyruvate carboxykinase (or PEPCK). Gluconeogenesis can then continue to run, and will generate glucose from these non-carbohydrate precursor molecules (choice B is correct). Although glyceraldehyde-3-P is a major intermediate in both gluconeogenesis and glycolysis, pyruvate and oxaloacetate cannot be converted directly into this molecule (choice A is incorrect). Both choices C and D are incorrect because they start with "No"; as discussed above, glucogenic amino acids can be converted into glucose (choices C and D are wrong).

Research indicates that professional burnout can result in thoughts of suicide. Suicidal ideation is most often associated with a psychological disorder in which broad category? A. Anxiety disorders B. Mood disorders C. Personality disorders D. Psychotic disorders

B. Thoughts of death or suicide are most often associated with major depressive disorder, which is a mood disorder characterized by persistent feelings of sadness and hopelessness (choice B is correct). Mood disorders involve disturbances in mood or affect. Anxiety disorders involve excessive worrying and include generalized anxiety disorder, panic disorder, and obsessive-compulsive disorder (choice A is wrong). Research does not show a significant association between these disorders and suicidal ideation, with the exception of post-traumatic stress disorder. Personality disorders involve enduring maladaptive patterns of behavior and cognition and include antisocial personality disorder, avoidant personality disorder, and borderline personality disorder (choice C is wrong). Psychotic disorders involve a loss of contact with reality and include schizophrenia and delusional disorder (choice D is wrong).

A person sees a small grassy area containing flowers outside of a residential building. This person's brain draws on various memories of gardens and the scents of flowers and grass in order to interpret that this grassy area is a garden. This is an example of: A. bottom-up processing. B. top-down processing. C. superimposition. D. contextual reception.

B. Top-down processing is a Gestalt psychology term for interpreting sensory stimuli based on experience and larger contextual information in order to slowly work down and figure out the precise nature of each stimulus (choice B is correct). Bottom-up processing is the Gestalt term for the reverse process, by which one starts with a small sensory stimulus and integrates progressively more contextual information in order to determine its nature (choice A is wrong). Superimposition is a process whereby one stimulus is layered on top of another (choice C is wrong). Contextual reception is simply another way of saying that one is receiving contextual stimuli, not necessarily even processing it (choice D is wrong).

Hate crimes committed against immigrants are a consequential example of: A. homophobia. B. xenophobia. C. a social phobia. D. a specific phobia.

B. Xenophobia is defined as the fear of that which is perceived to be foreign, such as the fear of cultural outsiders (choice B is correct). Xenophobic attitudes are one of the negative reactions to immigration, whether involving genuine physical fear or concerns about economic, political, or social factors (such as increased competition for jobs). Homophobia is more complex and is defined as the wide range of negative thoughts and attitudes about non-heterosexual individuals, as well as any resultant behavior (choice A is wrong). A social phobia is defined as the fear of potential embarrassment or humiliation in public situations (choice C is wrong). For example, the fear of using a public restroom is a common example of a social phobia that can influence one's ability to perform routine activities. A specific phobia is defined as the fear of a specific object or situation (choice D is wrong). These are divided into four categories based on the specific triggers: animal, medical, natural environmental, and situational phobias (e.g., arachnophobia is a common specific animal phobia of spiders).

In a facultative anaerobe, which of the following processes occurs under both aerobic and anaerobic conditions? A. Fermentation B. Krebs cycle C. Glycolysis Correct Answer D. Oxidative phosphorylation

C. A facultative anaerobe can survive through fermentation when oxygen is not available but will use oxidative respiration when oxygen is available. Glycolysis will occur under both aerobic conditions (in which case the pyruvate will go on to enter the Krebs cycle as acetyl-CoA) and under anaerobic conditions (fermentation reduces pyruvate to alcohol or lactate), so choice C is correct. Fermentation occurs only in anaerobic conditions (choice A is wrong), while the Krebs cycle and oxidative phosphorylation can occur only in aerobic conditions (choices B and D are wrong).

Which reinforcement schedule is defined as reinforcing a behavior after an unpredictable number of responses? A. Fixed-ratio B. Fixed-interval C. Variable-ratio D. Variable-interval

C. A variable-ratio schedule occurs when a participant receives a positive response or reward after an unpredictable number of responses (choice C is correct). A variable-interval schedule occurs when a participant receives a positive response or reward after an unpredictable amount of time (choice D is wrong). A fixed-ratio schedule rewards behavior after a specific number of responses (choice A is wrong). A fixed-interval schedule rewards behavior after a specific amount of time (choice B is wrong).

The episodic buffer allowed participants to: A. remember the instructions of the OLG task. B. form a visual picture of the video clip while recording response options. C. apply experience learned in prior games in order to generate options. D. avoid confusing the contents of one clip with those of another.

C. According to Baddeley, the working memory system is composed of four parts. The visuospatial sketchpad is a short-term visual store, the phonological loop is a short-term auditory store, the central executive is responsible for task-switching, and the episodic buffer integrates information from the other three systems and from long-term memory. An example of the contribution of the episodic buffer, therefore, is the recollection and application of prior game experience (choice C is correct). The episodic buffer is not directly responsible for the internal organization of experience (choice D is wrong). Rehearsal of verbal instructions would be the province of the phonological loop (choice A is wrong), while short-term visual memory of the video clips would be the province of the visuospatial sketchpad (choice B is wrong).

Researchers found that people with graduate or professional degrees were divorced less often than were those with other levels of education. Therefore, it is possible that the success of a marriage depends on: A. economic capital. B. political capital. C. cultural capital. = D. social capital.

C. According to social reproduction theory, inequality is transmitted from one generation to the next through various forms of "capital". Education is a classic example of cultural capital, which includes non-financial social assets such as knowledge, skills, and experience (choice C is correct). Education may be related to economic and political success; however, this is not supported by the information provided (choices A and B are wrong). The other choice, social capital, is also an important mechanism of social reproduction. It includes relationship-based social assets, like influence and "connections" with people in one's social network (choice D is wrong). It is important to note that the question stem states that cultural capital "might" determine marital success, as correlation does not necessarily imply causation.

A biochemist attempts to cleave a short peptide of known sequence into two fragments using proteinase K (a proteolytic enzyme). However when running the cleavage product on a gel, only a single peptide with no change in molecular weight is observed. What is the most likely cause of this observation? A. Incomplete cleavage of the phosphodiester backbone B. Running the cleavage product on a gel in a reducing environment C. Presence of multiple amino acids containing sulfur Correct Answer D. Proteinase K did not adequately stabilize the cleavage product

C. Enzymatic proteolytic cleavage results in the hydrolysis of a peptide backbone (choice A is wrong) at specific sites due to the stabilization of the transition state by the proteolytic enzyme (choice D is wrong). While this disrupts the primary structure of a protein, disulfide bonds can remain intact in oxidizing conditions and can result in the cleaved peptides remaining associated (choice B is wrong and choice C is correct).

During peak cardiac activity, heart muscle utilizes the anaerobic pathway for only 10% of its energy as opposed to up to 85% for maximally contracting skeletal muscle. With respect to skeletal muscle, cardiac muscle would: A. convert more pyruvate to lactate. B. consume 1/8 as much ATP to produce the same contractile force. C. consume less glucose per ATP produced. D. require less oxygen per molecule of ATP produced.

C. Aerobic respiration includes glycolysis, the PDC, the Krebs cycle, and electron transport, to produce a total of 32 ATP per glucose (assuming the malate-aspartate shuttle is used to transfer the electrons from glycolytic NADH into the mitochondria). By contrast, anaerobic respiration involves only glycolysis and fermentation of pyruvate to lactate or alcohol, to produce 2 ATP per glucose. Anaerobic respiration is therefore much less efficient and must consume much more glucose to produce the same amount of ATP. Since cardiac muscle uses mostly aerobic respiration, it will be more efficient than skeletal muscle and uses less glucose to produce an equivalent amount of ATP (choice C is correct). Conversion of pyruvate to lactate occurs during fermentation, which cardiac muscle carries out less of, not more (choice A is wrong). The amount of contractile force generated per ATP is independent of the source of ATP used (choice B is wrong). Cardiac muscle uses more aerobic respiration, so it will use more oxygen per ATP produced, not less (choice D is wrong).

Decreased secretion of aldosterone has all of the following effects EXCEPT: A. increased loss of Na + in urine. B. decrease in extracellular fluid volume. C. increase in arterial pressure. D. increase in urine volume.

C. Aldosterone causes increased sodium reabsorption, and because of the rise in systemic Na+, there will be decreased water loss, increased extracellular fluid volume and increased blood pressure. Decreased secretion of aldosterone would cause the opposite: increased loss of sodium (choice A is true and eliminated), a decrease in the extracellular fluid volume (choice B is true and eliminated), and an increase in urine volume (choice D is true and eliminated). It will not cause an increase in blood pressure, however, since more urine is formed and extracellular fluid volume lost (choice C is false and thus the correct answer choice).

Which of the following is an accurate statement concerning eukaryotes and prokaryotes? A. In most cases, eukaryotes follow the "one gene, one protein" rule, while prokaryotes are monocistronic. B. Eukaryotes have three DNA polymerases, and prokaryotes have three RNA polymerases. C. In prokaryotes transcription and translation can occur simultaneously, while in eukaryotes, they must remain distinct. D. Eukaryotes modify their primary transcripts by adding a 5' cap and a 3' poly-A tail, but prokaryotes only add a poly-A tail since the Shine-Dalgarno sequence replaces the function of the 5' cap.

C. Because prokaryotes do not have organelles, both transcription and translation occur in the cytoplasm, frequently simultaneously. In eukaryotes, however, transcription occurs in the nucleus, and translation occurs in the cytoplasm. Most eukaryotic RNA codes for a single protein with very few exceptions (alternative mRNA splicing); however, prokaryotic mRNA is often polycistronic and codes for several different proteins, often by utilizing different reading frames (A is wrong). Eukaryotes and prokaryotes both have multiple DNA polymerases (more than three), but eukaryotes have three RNA polymerases while prokaryotes have only a single RNA polymerase (B is wrong). While it is true that the Shine-Dalgarno sequence in prokaryotes essentially replaces the function of the 5' cap in eukaryotes, prokaryotes do not add poly-A tails to their mRNA transcripts (D is wrong).

The most likely explanation for the inverse relationship between mammal size and pulse is that: A. most small mammals live under anaerobic conditions. B. large animals tend to consume proportionately more food than smaller animals. C. smaller mammals have a higher metabolic rate for a given mass of tissue than larger mammals. D. smaller mammals have reduced oxygen requirements for a given mass of tissue.

C. Choice A is wrong since all mammals use primarily aerobic respiration except for short periods of unusually strenuous activity. Choice B is wrong since it does not directly involve metabolic rate or pulse and would predict that the metabolic rate and pulse rate will remain the same despite size. Choice D is wrong since reduced oxygen requirements would predict a slower pulse. Choice C is the best choice: If tissues in smaller animals have a higher metabolic rate, they will require a greater supply of blood to provide oxygen and nutrients and carry away carbon dioxide. To meet these greater needs, the pulse rate could be increased.

Eukaryotic cells use reciprocal regulation to make sure β-oxidation and fatty acid biosynthesis do not occur at the same time. This is accomplished by malonyl CoA, which inhibits the carnitine shuttle required to transfer activated fatty acids from the cytoplasm to the mitochondrial matrix. Each of the following is a true statement EXCEPT: A. both β-oxidation and fatty acid biosynthesis occur in four steps; fatty acid biosynthesis involves elongation, two redox reactions and a dehydration. B. β-oxidation involves the reduction of both FAD and NAD+; fatty acid biosynthesis oxidizes two NADPH (generated by the pentose phosphate pathway) to two NADP+. C. because of this reciprocal regulation, both β-oxidation and fatty acid biosynthesis occur in the mitochondrial matrix. D. β-oxidation generates acetyl CoA, while fatty acid biosynthesis uses malonyl CoA, which is made from acetyl CoA by acetyl CoA carboxylase.

C. Choices A, B and D are true statements. Choice C is false (and the correct answer); β-oxidation occurs in the mitochondrial matrix and fatty acid biosynthesis occurs in the cytoplasm.

Which of the following conditions leads to a reduction in water reabsorption by the kidney? A. High plasma levels of vasopressin B. Increased osmolarity of interstitial fluid in the kidney medulla C. Decreased permeability of the collecting duct to water D. An ACTH-secreting tumor

C. Factors that reduce water resorption would tend to increase urinary output, decrease blood volume, and decrease blood pressure. High levels of vasopressin (ADH) would increase water resorption and plasma volume (choice A is wrong). The greater the osmolarity of the interstitial fluid in the medulla of the kidney, the greater the water resorption as filtrate passes into the medulla in the descending loop of Henle (choice B is wrong). As filtrate passes through the collecting duct, water can be reabsorbed and the urine concentrated. If the collecting duct is impermeable to water, however, then less water will be reabsorbed, and the filtrate will remain dilute (choice C is correct). ACTH is the hormone that controls secretion of aldosterone (as well as cortisol) by the adrenal cortex. An ACTH-secreting tumor will cause elevated secretion of aldosterone, increased Na+ reabsorption, and a subsequent increase in water reabsorption (choice D is wrong).

If one woman in a small and isolated Korean village fell ill with shin-byung, and then her close friend also began experiencing similar symptoms, claiming that she too was being pursued by spirits, which of the following might explain this occurrence regarding the friend? A. Compliance followed by obedience B. Conformity followed by compliance C. Conformity followed by private acceptance D. Obedience followed by conformity

C. Conformity, obedience, and compliance are related but distinct terms. Conformity involves changing one's behavior in order to fit in with the norms of a particular social group, most typically a group that has a certain level of social importance. Obedience involves changing one's behavior in response to the direct command or order of a person who is in authority or is of higher social status. Compliance involves changing one's behavior in response to a request from another person who is of equal or lower status (thus differing from obedience). Private acceptance is an attitude change that can occur in a person due to the social influence of others. Thus, it is most likely that the friend first conformed to the behavior of the woman initially affected by shin-byung symptoms, then privately accepted the possibility of spirit possession after modifying her behavior (choice C is correct). One could comply with a request from a peer and then obey a command from a leader, but that would not explain the situation described (choice A is wrong). Similarly, changing one's behavior in accordance with a group norm and then responding to a request from a peer would not fit the described situation either (choice B is wrong). Finally, following the orders of an authority figure and then the norms of a desired social group would not explain the shin-byung symptoms in this scenario (choice D is wrong).

Suppose that Sarah is kneading a piece of clay in her hands. Which of the following is NOT a strategy relevant to conservation? A. Sarah realizes that the clay is the same piece of clay, regardless of its shape. B. Sarah notices that she can shape the clay into a ball, then a pancake, then back into a ball again. C. Sarah knows that the clay still exists even when her mother puts it back into its container. D. Sarah discovers that squeezing the clay in the middle forces it to become bulkier on the sides.

C. Conservation is the term that Piaget used to describe children's recognition of constancy and consistency in physical matter despite changes in shape or container. This awareness is characteristic of the stage of concrete operational thought, which is usually achieved between the ages of 6 or 7 and 11 or 12. Conservation is realized through three insights: identity, reversibility, and reciprocity. Identity involves the child's recognition that the physical object is the same object, regardless of how it is manipulated (choice A is a strategy relevant to conservation and is therefore wrong). Reversibility involves the child's recognition that a given manipulation can be reversed to give the object its initial appearance (choice B is a strategy relevant to conservation and is therefore wrong). Finally, reciprocity involves the child's realization that a manipulation of one dimension or aspect yields a corresponding change in another aspect. As a result, the manipulation is understood to change the original object rather than create a new one (choice D is a strategy relevant to conservation and is therefore wrong). The realization that an object continues to exist despite its disappearance from the visual field is known as object permanence (choice C is not a strategy relevant to conservation and is therefore correct).

Metformin is used in the treatment of diabetic patients. One of the possible effects of metformin on hepatocytes will be to: A. stimulate gluconeogenesis to decrease blood glucose levels. B. stimulate glycolysis to increase blood glucose levels. C. inhibit gluconeogenesis to decrease blood glucose levels. D. inhibit glycolysis to decrease blood glucose levels.

C. Diabetics have increased glucose levels in their blood. Therefore, it is expected that the actions of metformin will be to lower the levels of glucose in the blood. Stimulating gluconeogenesis would increase blood glucose levels (choice A is wrong). Stimulating glycolysis will result in decreased blood glucose levels, while inhibiting it will result in increased blood glucose levels, not the other way around; and in any case, diabetics are unable to take up glucose and thus could not run glycolysis at all (choices B and D are wrong). Inhibiting gluconeogenesis will decrease the glucose production by the liver and hence decrease the blood glucose levels, making C the correct answer choice.

All of the following processes occur during the conduction of a nerve impulse across the synapse of two neurons EXCEPT: A. the release of vesicles from the presynaptic cell. B. the opening of postsynaptic ion channels. C. a retrograde depolarization of the presynaptic cell. D. the depolarization of the postsynaptic cell.

C. During the conduction of an action potential across a chemical synapse, neurotransmitter is released by the presynaptic cell, it diffuses across the synapse, binds to receptors on the postsynaptic cell, opens ion channels on the postsynaptic cell, and depolarizes (or hyperpolarizes) the postsynaptic cell. Thus, choices A, B, and D are all part of the normal conduction of an action potential across a synapse and can be eliminated. The presynaptic cell does not normally transmit a retrograde action potential, however (choice C is false and the correct response here). This is usually prevented by the refractory period after an action potential passes, causing action potentials to propagate in only one direction along an axon.

Theorists using the conflict perspective to address questions of health, illness, and medicine are LEAST likely to consider: A. the commodification of health care delivery. B. the presence of disparities in health and health care. C. the power of professionals in patient-provider relationships. D. the dependence of medical institutions on profit.

C. Each of the main sociological perspectives—structural functionalism, conflict theory, and symbolic interactionism—is expected to approach health and health care differently. The conflict theory approach considers the competition for limited resources. According to this theory, limited resources, a negative result of capitalism, cause inequalities and power differentials in society. This concept can be extended to the medical institution. Furthermore, Karl Marx, the founding conflict theorist, focused on the internal tensions produced in capitalistic societies. Thus, it is expected that the conflict perspective would consider the commodification of health care and the dependence of institutions on profit (choices A and D can be eliminated). Health care can thus be purchased and sold in a marketplace, which allows those with resources and influence to control pertinent decisions, thus maintaining inequalities. Furthermore, these social inequalities determine the distribution of health care access and resources. Thus, it is expected that the conflict perspective would consider the resultant disparities in health and health care (choice B can be eliminated). However, while conflict theorists are interested in the creation and maintenance of power differentials, as proponents of a macro-sociological perspective, this interest is applied to social structures as a whole. The patient-provider relationship would perhaps be of more interest to symbolic interactionists, as symbolic interactionism is a micro-level perspective focused on the transfer of information through communication between individuals (choice C is not an expected focus of conflict theorists and is the correct answer). Furthermore, it is possible that a power differential is inherent in the provider-patient relationship as a result of the expertise required for medical professionals, thus making an egalitarian relationship difficult to create.

Glucose-6-phosophate dehydrogenase (G6PDH) deficiency is an inheritable metabolic disorder that can result in the destruction of red blood cells, and in severe cases, can lead to kidney and liver failure. Which of the following best describes the reason why G6PDH deficiency can result in premature red blood cell destruction? A. G6PDH deficiency results in a buildup of glucose-6-phosphate, increasing the solute concentration in the intracellular environment, causing the cell to swell and lyse. B. G6PDH deficient cells are recognized as abnormal and are targeted for destruction by macrophages. C. G6PDH deficiency results in a deficiency of NADPH, which results in increased damage from reactive oxygen species and causes premature cell death. D. G6PDH deficiency results in an increased intracellular pH, which denatures proteins and triggers apoptosis.

C. Glucose-6-phosphate dehydrogenase (G6DPH) catalyzes the first step in the pentose phosphate pathway, where NADPH is produced. NADPH plays a major role in preventing damage due to oxidative stress/reactive oxygen species. A deficiency of G6PDH prevents the production of NADPH, thus individuals with this deficiency would experience increased damage from reactive oxygen species (choice C is correct). G6PDH deficiency would not be expected to alter the intracellular osmolarity (choice A is incorrect) or to increase the intracellular pH (choice D is also incorrect). Choice B is incorrect: macrophages are not involved in the death of red blood cells in G6PDH cells.

Glycogen is a highly branched polymer of glucose which aids in maintaining blood glucose levels in a fasted state. Which of the following is NOT true of glycogen breakdown? A. It involves cleavage of α-(1→6) glycosidic linkages. B. It involves cleavage of α-(1→4) glycosidic linkages. C. It involves cleavage of glycolytic intermediates from the glycogen polymer. D. Glucagon has a limited effect on glycogenesis in skeletal muscle.

C. Glycogenolysis results in the cleavage of a molecule of glucose-1-phosphate from the polymer which must be converted to glucose-6-phosphate by phosphoglucomutase in order to enter glycolysis (choice C is not true of glycogenolysis and is the answer). Glycogen possesses both α-(1→4) and α-(1→6) glycosidic linkages and is present predominantly in the liver where it is released when fasting (choices A and B are wrong). A smaller quantity of glycogen is stored in skeletal muscle but serves as a readily releasable source of fuel response to epinephrine, not glucagon (choice D is wrong).

Suppose that a team of six doctors who are colleagues at a university has conducted a study about a new medication for the common cold. Four of the researchers deem the medication effective and agree that it should be available in a non-prescription form. The two remaining doctors, however, are concerned about the drug's potentially fatal side effects when taken in large doses. In the end, the two dissenting doctors conform to the opinion of the other four and agree that the medication should be available without a prescription. This is an example of: A. group polarization. B. the bystander effect. C. groupthink. D. social facilitation

C. Groupthink involves the notion that members of groups, especially those groups that value cohesiveness among members, will conform in order to eliminate disharmony, which leads to a faulty decision-making process. In the above scenario, the research team members are colleagues at a university and likely have an ongoing professional relationship; accordingly, they may feel a strong need to maintain group harmony and unity, perhaps with fatal consequences for patients (choice C is correct). Group polarization occurs when, after pertinent discussion or argument, a group adopts a more extreme position than the positions to which the individual members adhere. In this case, the group did not develop a more extreme position than the one advocated by the original four concurring team members; the remaining two doctors simply deferred to their colleagues (choice A is wrong). The bystander effect demonstrates that the likelihood that an individual in need will receive assistance decreases as the number of individuals present increases. One of the most famous examples of the bystander effect involved Kitty Genovese, a woman who was brutally murdered while her neighbors did nothing to intervene (choice C is wrong). Social facilitation refers to the idea that an individual who is familiar with a task will perform better in the presence of others (choice D is wrong).

Which of the following amino acid residues would likely be found on the protein interior for proteins in aqueous solution? Alanine Valine Lysine Glutamate A. I, II, and IV B. II and IV C. I and II D. II and III

C. In aqueous solution, we expect hydrophilic or charged residues to be found on the protein exterior near the polar water and the hydrophobic, nonpolar residues to be facing the protein interior. Alanine and valine are both hydrophobic residues. Lysine is a charged basic residue while glutamate is a charged acidic residue. Given that, answer choice C, alanine and valine are expected to be on the protein interior.

Which of the following is/are true regarding Gestalt laws of perceptual organization? Certain basic features of vision, including color and motion, are processed consciously in a bottom-up manner In our visual field, we tend to group nearby objects together; this is known as "proximity" When viewing something, we tend to fill in gaps in order to complete whole objects; this is known as "closure" A. II only B. III only C. II and III only D. I, II, and III

C. Item I is false: basic visual features, such as color and motion, are processed instantly and automatically by the brain, not consciously in a bottom-up fashion (choice D can be eliminated). Item II is true: one of the Gestalt principles of grouping is that of proximity; we tend to group nearby figures or objects together (choice B can be eliminated). Item III is true: another of the Gestalt principles of grouping is that of closure; we tend to fill in gaps in order to perceive objects that are whole and complete (choice A can be eliminated and choice C is correct).

There are multiple sociological perspectives on deviance. Which of the theorists below are expected to consider the associated questions about non-normative behaviors? Differential association theorists; "How can people resist deviance?" Labeling theorists; "Who defines deviance?" Structural strain theorists; "How do norms affect deviance?" A. III only B. I and II only C. II and III only D. I, II, and III

C. Item I is false: differential association theorists argue that deviant behaviors are learned through interactions between individuals and their communities. These theorists contend that people become deviant when they are exposed to deviant behavior and attitudes. "How can people resist deviance?" is therefore not a pertinent question (choices B and D can be eliminated). The main criticism of differential association is that it essentially reduces individuals to their environments. Because it states that those in deviant communities learn to be deviant themselves, the possibility of resistance is not contemplated. Item II is true: labeling theory asserts that behaviors are seen as deviant as the result of social processes of labeling. "Who defines deviance" is therefore a pertinent question (choice A can be eliminated). Labeling theorists do indeed address this concern; for example, these theorists are interested in the mechanisms through which power contributes to deviance labeling (e.g., agents of social control). Item III is true: structural strain theory suggests that deviant behaviors are the result of tension between the accepted social goals and the institutionalized means available to achieve those goals. "How do norms affect deviance?" is therefore a pertinent question (choice C is correct). Structural strain theorists consider the effects that social norms have on behavior, normative or not, and suggest that there is pressure to use deviant methods when the social structure does not support the achievement of the accepted goals.

Penicillin is an antibiotic that interrupts the synthesis of bacterial cell walls. Which of the following is most likely to be affected by its action? A. Fatty acids B. Phospholipids C. Peptidoglycans D. Lipoproteins

C. Since bacterial cell walls are made up of proteins and carbohydrates (peptidoglycans), if penicillin affects cell wall synthesis it will most directly affect peptidoglycan (C is correct). Fatty acids and phospholipids are parts of cell membranes (A and B are wrong), and lipoproteins are blood proteins that transport lipids (D is wrong).

Suppose that a young unwed teenager who dropped out of high school and frequently uses drugs gets pregnant and has a daughter. In high school this daughter begins to associate with a crowd that uses drugs, experiments with drugs herself, has unprotected sex, and is on the verge of getting expelled from school. Which of the following best explains what might be occurring in this scenario? Downward mobility Intergenerational mobility Social reproduction A. I only B. II only C. III only D. II and III only

C. Item I is false: downward mobility describes a lowering in social class for an individual. There is not enough information to ascertain that the social class of either the mother or the daughter has declined. In fact, since the daughter is behaving in ways similar to how her mother behaved at her age, it might be inferred that the social class of both individuals has remained the same (choice A can be eliminated). Item II is false: intergenerational mobility occurs when there is a change in social class between parents and children within a family; again, there is not enough information to ascertain if this is the case, and, if anything, it appears as though the mother and daughter are similar in terms of social class (choices B and D can be eliminated). Item III is true: social reproduction occurs when structures and activities in place within a society serve to transmit and reinforce social inequality from one generation to the next. Cultural and social capital are two mechanisms by which social reproduction occurs. Cultural capital (such as education) affords one the potential to be upwardly mobile, as does social capital (one's social network). Since the daughter is not utilizing either cultural capital or social capital to her advantage, she seems to be making some of the same choices that her mother made. Therefore, social reproduction best explains this scenario (choice C is correct).

In-group favoritism involves which of the following: prejudice against an out-group. discrimination against an out-group. discrimination against an in-group. A. II only B. III only C. I and II only D. I and III only

C. Item I is true: prejudice involves negative biases about members of a group. Since in-group favoritism involves favoring members of an in-group over members of an out-group, in-group favoritism involves prejudice against the out-group (choices A and B can be eliminated). Item II is true: discrimination involves unjust actions that result from prejudice. In-group favoritism involves actions that favor one group of people over another (choice D can be eliminated). Item III is false: in-group favoritism involves preferential treatment of the in-group rather than discrimination against the in-group (choice C is therefore the correct answer).

What is the minimum number of moles of CaSO4(s) that must be added to one liter of water in order to create a saturated solution if the Ksp of CaSO4(s) is 9.1 x 10-6? A. 9.10 x 10-3 B. 1.74 x 10-6 C. 3.02 x 10-3 D. 3.02 x 10-6

C. Ksp = [Ca2+][SO42-], and the molar solubility, or the number of moles of CaSO4 that can dissolve to make a saturated solution, is the square root of Ksp, since Ksp = (x)(x) = x2. Quickly eliminate choices B and D by looking at the power of 10. Since the square root of 9 is 3, the correct answer is C.

Measurements of which of the following would be sufficient to calculate the usable work performed by a mechanical engine? A. The rate at which energy is consumed by the engine and the amount of time the engine was running B. The heat produced by the engine and the amount of time the engine was running C. The power output of the engine and the amount of time the engine was running D. The energy consumed by the engine and the amount of time the engine was running

C. Power is defined as work divided by time: P = W / t. Therefore, if the power output P and the time t are known, the work performed can be calculated by the equation W = Pt. Since we want the usable work produced by the engine, which presumably is not 100% efficient, the output power is needed, which will be less than the input power.

Which of the following would be the most effective method to determine if transcription of a gene had been silenced? A. DNA fingerprinting B. ELISA C. RT-PCR D. DNA sequencing

C. RT-PCR uses reverse transcriptase to make cDNA from mRNA. The cDNA can then amplified, and then tested for. This is the only method listed that can be used to test for the presence of mRNA and determine if transcription of the gene is occurring or not (choice C is correct). Both DNA Fingerprinting and DNA Sequencing are methods used to analyze DNA sequences; this does not provide information about transcription (choices A and D can be eliminated). While ELISA could be used to test for the presence of a protein product, it would not be the best test to use in this situation (choice B is incorrect). The absence of a protein could be due to a variety of reasons, including problems with translation, protein degradation, etc. Thus, RT-PCR would be a better method to specifically test for gene expression (i.e., transcription).

Which of the following statements primarily differentiates between schizophrenic behavior and schizoid affect? A. Schizophrenics hear voices and schizoids have delusions. B. Schizophrenics have delusions and schizoids have thought disorders. C. Schizophrenics exhibit psychotic behaviors and schizoids withdraw from interactions with others. Correct Answer D. Schizophrenics exhibit flat affect and schizoids have bursts of wild behavior.

C. Schizophrenia is classified as a brain disorder that may include any combination of the following symptoms: positive symptoms (familiarly known as psychotic symptoms such as delusions, hallucinations, thought disorders, or movement disorders) and negative symptoms (a flat affect, a loss of pleasure in everyday life or previously pleasurable activities, and a lack of interest in beginning or continuing with activities or in interacting with others). Cognitive symptoms include poor executive functioning (e.g., decision-making) and trouble with focusing or remembering (choices A, B, C, and D all accurately describe schizophrenic symptoms). "Schizoid" refers to a personality disorder, which is a different diagnostic category, and is characterized primarily by an unwillingness to interact with others. Those with schizoid personality disorder may seem aloof or to be loners, and may or may not prefer that state of being (choice C is correct). Typically schizoid individuals do not have delusions (choice A is wrong), do not have thought disorders (choice B is wrong), and do not demonstrate bursts of wild behavior (choice D is wrong).

A secure attachment style is best demonstrated by which of the following scenarios? A. A five-year-old child who remains by her mother's side at a children's birthday party, not interacting with the other children B. A four-year-old child who runs toward her mother with open arms upon her return from work, but before reaching her turns around and instead runs away frightened. C. A four-year-old who confidently joins in a wide array of activities while at a birthday party and later seeks his mother's help, and is soothed by her, when a game of tag results in a skinned knee D. A five-year-old child who explores different parts of a playground, pushing his climbing of the jungle gym to dangerous heights, with the knowledge that his mother will not be paying attention

C. Secure attachment is promoted by a responsive and available parent and is characterized by a child who is able to explore his or her environment secure in the knowledge that the parent will be there when needed. A child confidently exploring and joining in different activities at a birthday party demonstrates a secure attachment style. The child's act of seeking his mother's aid during a stressful time, such as when being injured, and the parent being available to comfort and soothe him, further displays secure attachment (choice C is correct). Reluctance to explore and instead clinging and remaining by the mother's side is indicative of an anxious-resistant insecure attachment style, in which the child is uncertain about the parent's availability because of previous separations. Because of this fear of being abandoned, he or she stays close and clings to the mother (choice A is wrong). Odd or awkward behaviors when separating or reuniting with the parent, such as extreme shifts between proximity-seeking and avoidance, are indicative of a disorganized attachment style. A child who, upon being reunited with her mother, initially runs toward her with open arms and then quickly runs away instead illustrates an extreme shift between proximity-seeking and avoidance. Disorganized attachment is usually the result of the child experiencing trauma, such as being physically, emotionally, or sexually abused by the parent. In this instance, the child sees the parent as both frightening and the only source of comfort, which causes great confusion and results in the eratic proximity-seeking and avoidance behavior (choice B is wrong). An anxious-avoidant attachment style results from the child being constantly rejected and rebuffed by the parent. These children learn to be independent early on, depending only on themselves because they learn that they cannot rely on the parent who is never available. A child exploring and climbing the jungle gym to dangerous heights displays this early independence. The child's knowledge about the lack of availability and responsiveness of the parent further provides evidence of an anxious-avoidant attachment style (choice D is wrong).

Which of the following is NOT true regarding somnambulism (sleepwalking), a type of parasomnia? A. Sleepwalking tends to occur during slow-wave sleep B. Typically individuals will engage in sleepwalking during the first third of the night C. Individuals will often act out the dreams they are concurrently having while sleepwalking D. While somnambulism tends to affect many children, most will grow out of it by adulthood

C. Somnambulism, or sleepwalking, occurs during slow-wave non-REM sleep (choice A is wrong), typically during the first third of the night (which corresponds to the fact that more slow-wave, deep sleep occurs early in the night; choice B is wrong); therefore, since it occurs during non-REM sleep, it is not possible for sleepwalkers to be concurrently dreaming (which only occurs during REM sleep), so sleepwalkers cannot be acting out the dreams they are having (choice C is correct). Somnambulism is common in children and studies suggest that up to 20-25% of all children have experienced at least one sleepwalking episode. Sleepwalking usually disappears by adolescence (choice D is wrong).

Which of the following is true regarding the specific enzymes involved in DNA replication? A. DNA topoisomerases function to produce supercoils during DNA replication. Your Answer B. DNA polymerase can only initiate DNA replication following the placement of a RNA primer by the enzyme DNA primase. C. DNA ligase functions by catalyzing phosphodiester bond formation between the ends of two strands of DNA. D. Telomerase preserves genetic information found in DNA by adding a sequence of DNA onto the 5' ends of DNA strands.

C. The Okazaki fragments produced during DNA replication must be joined by DNA ligase with the formation of a phosphodiester bond (choice C is correct). DNA topoisomerases create cuts at portions of the DNA helix to remove supercoils created during replication (choice A is wrong). The enzyme responsible for creating the RNA primer is RNA primase (choice B is wrong). Telomerase adds DNA to the 3' ends of DNA strands (choice D is wrong).

What is the main premise of the behaviorist theory of personality? A. Personality is the result of a person's multiple traits. B. Personality is the result of a person's unconscious mind and childhood experiences. C. Personality is the result of interactions between the person and the environment. D. Personality is the result of a person's genetics.

C. The behaviorist theory of personality emphasizes that personality is derived from the interactions between a person and his or her environment (choice C is correct). Trait theory, a type of humanist theory, emphasizes that personality is the result of a person's multiple traits that are relatively stable over time (choice A is wrong). Psychodynamic theories emphasize that personality is the result of a person's unconscious mind and childhood experiences (choice B is wrong). Biological theories of personality emphasize that personality is the result of a person's genetics (choice D is wrong).

The vestibular sense detects motion and is involved in balance by way of what mechanism? A. Control of autonomic functions by the pons B. The endolymph in the cochlea contains hair cells that detect motion and rotation of the head C. The endolymph in the utricle, saccule, and ampullae contain hair cells that detect motion and the rotation of the head D. Coordination of motion in cerebellum

C. The endolymph in the utricle, saccule, and ampullae—the semicircular canal of the inner ear—contain hair cells that detect motion and the rotation of the head. This aids in the sense of balance (choice C is correct). The pons in the hindbrain controls some autonomic functions and coordinates movement and thus is related to the sense of balance but that control of autonomic functions on its own is not the vestibular sense (choice A is wrong). The endolymph in the cochlea contains hair cells that are designed to transmit auditory vibrations for the sense of hearing, not proprioception or balance (choice B is wrong). The cerebellum coordinates complex movements and damage to that area results in difficulties in movement and balance, but again this is not the specific mechanism of the vestibular sense (choice D is wrong).

Which of the following accurately lists the composition of the endoplasmic reticulum, phosphofructokinase and alanine, respectively? A. Double membrane surrounding a lumen, amino acids linked together by covalent proteolytic bonds, four different groups on a chiral Alpha carbon. B. Single membrane surrounding a lumen, amino acids linked together by hydrogen bonds, four different groups on a chiral Alpha carbon. C. Single membrane surrounding a lumen, amino acids linked together by covalent peptide bonds, four different groups on a chiral Alpha carbon. D. Single membrane surrounding a lumen, amino acids linked together by covalent peptide bonds, three different groups on a chiral Alpha carbon.

C. The endoplasmic reticulum is an organelle with a single membrane surrounding a lumen (eliminate choice A). Phosphofructokinase is an enzyme of glycolysis, and so is a protein made of amino acids linked together by covalent peptide bonds (choice B is wrong). Alanine is a chiral amino acid, and so has four different groups on a chiral alpha carbon (another reason to eliminate choice D). Choice C provides an accurate description and is correct.

Which of the following represents the correct sequence for lytic DNA bacteriophage replication? A. Genome transcription, infection of the host cell, mRNA translation, progeny assembly B. Infection of the host cell, mRNA translation, genome transcription, progeny assembly, genome replication C. Infection of the host cell, genome transcription, mRNA translation, progeny assembly D. Infection of the host cell, genome replication, mRNA translation, genome transcription, progeny assembly

C. The first step in any viral life cycle is infection (A is wrong). Since the question asks about a DNA virus, the next step must be transcription of the viral genome into mRNA (B and D are wrong, and C is correct).

The melting temperature of the 15-mer oligonucleotide 5'-GACTGCGGTTTTAAC-3' is 42 °C. Which of the following oligonucleotides would be found primarily in a random coil state at 42 °C? A. 5'-GACTGCGGCGCGAAC-3' B. 5'-GCCTGAGGTTTTAGC-3' C. 5'-GACTGTAATGGTAAC-3' D. 5'-GACTGCGGCCCCAAC-3'

C. The melting temperature is the temperature at which half of the DNA strands are in the random coil or single-stranded state. As the given oligonucleotide has a melting temperature of 42°C, then the correct answer must be an oligonucleotide with a melting temperature with a lower melting temperature in order to be primarily in the random coil state at 42°C. In order to have a lower melting temperature, the oligonucleotide must contain fewer GC nucleotides than the given oligonucleotide (Choice C is correct; Choices A, B, and D are wrong). This is because GC base pairing involves three hydrogen bonds while AT only involves two.

Hydrolysis of the phosphocreatine bond releases 43.1 kJ/mol of energy. What is the standard free energy of hydrolysis for ATP? A. -87.4 kJ/mol B. -43.1 kJ/mol C. -30.5 kJ/mol D. +6.7 kJ/mol

C. The passage says that creatine kinase can transfer a phosphate group from phosphocreatine to ADP via a spontaneous reaction. In other words, phosphocreatine dephosphorylation can be coupled to ADP phosphorylation and the net overall reaction is spontaneous, or has a -ΔG. If phosphocreatine dephosphorylation releases 43.1 kJ/mol, ADP phosphorylation must use less energy than this (choice C is correct). If ATP hydrolysis released 87.4 kJ/mol or 43.1 kJ/mol, the transfer of a phosphate group from phosphocreatine to ADP would not be a spontaneous reaction (choices A and B are wrong). ATP hydrolysis releases energy to the cell, so must have a negative standard free energy (choice D is wrong).

The process by which the nitrogen metabolism by-product urea is removed from the blood in the glomerulus is known as: A. tubular secretion. B. reabsorption. C. ultrafiltration. D. osmosis.

C. The passage states that filtration occurs at the glomerulus in Bowman's capsule. This allows small substances to travel into the proximal convoluted tubule. Tubular secretion occurs for ions such as K+ in the distal convoluted tubule (A is incorrect). Reabsorption is the process by which substances in the tubular filtrate are brought back into the blood (B is incorrect). Osmosis refers to the movement of water down its concentration gradient (D is incorrect).

Which of the following salts, when dissolved in water, will decrease the pH of the solution? NH4Cl KHSO4 NaHCO3 A. I only B. II only C. I and II only D. I, II, and III

C. The question is asking for the acidic salt(s). The conjugate acid of a weak base will lower the pH, while the conjugate base of a weak acid will raise it. Conjugates of strong acids or bases have no effect on pH. NH4+, the conjugate of the weak base NH3, is acidic. Cl-, the conjugate of the strong acid HCl, will not affect pH. Thus, Item I is an acidic salt (eliminate choice B). K+ and HSO4- are conjugates of a strong base and strong acid, respectively. Generally this would indicate a neutral salt. However, the HSO4- ion is acidic in its own right and will lower the pH, so Item II is also true (eliminate choice A). Na+ is the conjugate of a strong base and HCO3- is the conjugate of a weak acid, so Item III is a basic salt and will raise the pH of the solution (eliminate choice D).

Which of the following will bring about an increase in the ratio of separated charge stored on the plates to the potential difference between the plates of a parallel-plate capacitor? An increase in the distance between the plates An increase in the area of the plates An increase in the dielectric constant of the material between the plates A. I and II only B. I and III only C. II and III only D. I, II, and III

C. The ratio of the separated charge stored on the plates to the voltage between them, Q / V, is the definition of the capacitance, C. If a dielectric is in place, the capacitance of a parallel-plate capacitor is given by the equation C = K(ε0A /d), where K is the dielectric constant, A is the area of each plate, d is the distance between the plates, and ε0 is a constant. Therefore, increasing A (Item II) or increasing K (Item III) will increase C, but increasing d (Item I) will cause a decrease in C.

Which of the following infant reflexes in most closely associated with learning to balance and walk? A. Moro reflex B. Babinski reflex C. Stepping reflex D. Rooting reflex

C. The stepping or walking reflex involves a baby putting one foot in front of the other when their soles touch a flat surface; this reflex goes away around 8 weeks of age and resurfaces several months later when the baby is actually learning to walk (choice C is correct). The Moro reflex, also known as the startle reflex, is present from birth to about 4 or 5 months of age, and involves the infant spreading arms out then retracting them and usually crying in response to being startled (choice A is wrong). The Babinski reflex, which is normal in infants and disappears by 12-24 months, involves the fanning out of toes and dorsiflexion of the big toe in response to stroking the sole of the foot. The Babinski reflex, if present in an adult, can signify disease in the spinal cord or brain. This reflex is not associated with balance and learning to walk (choice B is wrong). The rooting reflex, present from birth to about 4 months of age, involves an infant turning it's head and rooting with it's mouth in response to a touch to the cheek or lips (choice D is wrong).

A therapist uses a white noise machine in her office for privacy; the therapist has been using the machine for years and doesn't even notice the sound it makes when it is on. One of her patients finds that noise produced by the white noise machine very irritating; every time he hears it he becomes more and more agitated. The therapist: A. has become sensitized to the white noise while the patient has become desensitized to it. B. and the patient have both been conditioned to the noise, but with different outcomes. C. has become habituated to the white noise while the patient has become sensitized to it. D. has generalized the white noise while the patient has discriminated it.

C. This example demonstrates non-associative learning. Habituation involves becoming so used to a stimulus that it no longer produces a response; the therapist no longer even noticing the sound produced by the machine is an example of habituation. Sensitization involves becoming more responsive to a stimulus; the patient demonstrating increased aggravation by the noise is an example of sensitization (choice C is correct). Desensitization involves trying to no longer respond to a stimulus that one was once sensitized to; the patient is currently sensitized to the stimulus, and would have to engage in an active process in order to become desensitized to it (choice A is wrong). This scenario demonstrates non-associative learning; conditioning is a type of associative learning (choice B is wrong). Generalization (responding with the conditioned response to stimuli other than the conditioned stimulus) and discrimination (responding with the conditioned response to only the conditioned stimulus) are both aspects of classical conditioning, a form of associative learning (choice D is wrong).

Which of the following best describes the cascade that leads to cellular apoptosis after exposure to intra- or extracellular death signals? A. Effector kinases cluster together and activate each other. The activation of effector kinases leads to the activation of initiator caspases, which cleave proteins at aspartic acid sites, triggering apoptosis. B. Effector caspases cluster together and activate each other. The activation of effector caspases leads to the activation of initiator caspases, which cleave proteins at aspartic acid sites, triggering apoptosis. C. Initiator caspases cluster together and activate each other. The activation of initiator caspases leads to the activation of effector caspases, which cleave proteins at aspartic acid sites, triggering apoptosis. D. Effector kinases cluster together with initiator capsases, activating the initiator capsases. The activation of effector kinases leads to the deactivation of initiator caspases, which cleave proteins at aspartic acid sites, triggering apoptosis.

C. This is a recall question pertinent to concepts discussed in the passage (apoptosis). Apoptosis is carried out by proteases called caspases, which cleave targets at aspartic acid residues; apoptosis is not started by effector kinases (eliminate choices A and D). Initiator caspases cluster together and initiate apoptosis (choice C is correct). These initiator caspases can then activate effector caspases, which cleave proteins at aspartic acid residues; this triggers apoptosis. Choice B is incorrect, and confuses the role of initiator and effector caspases.

At physiological pH, the amino acid with the formula C4H7NO4 has: A. two deprotonated carboxylic acid groups and is ketogenic. B. one deprotonated carboxylic acid group and is ketogenic. C. two deprotonated carboxylic acid groups and is glucogenic. Correct Answer D. one deprotonated carboxylic acid group and is glucogenic.

C. This is a typical two-by-two question. Since none of the pictured amino acids in Figures 1 or 2 have four oxygen atoms, the amino acid with the formula C4H7NO4 must be glucogenic (choices A and B can be eliminated). Only aspartic acid and glutamic acid have four oxygen atoms, two of them found in the carboxylic acid end of the backbone, and two of them found in the carboxylic acid end of the side chain, so this question must be asking about one of these amino acids. In actual fact, aspartic acid has the formula C4H7NO4, but you don't need to figure this out to answer the question. At physiological pH (about 7.4), both the carboxylic acid end of the amino acid and the side chain COOH will be deprotonated. This is because physiological pH is above the pKa values for these acidic protons (carboxyl pKa is approximately 2, and the R-group pKa is approximately 4). Thus, aspartic acid will actually be in its aspartate form, and will have two deprotonated carboxylic acid groups at physiological pH (choice C is correct and choice D is wrong).

Many forces influence how people relate to one another, become socialized beings, and learn to develop relationships. Which of the following is NOT one of the agents of socialization? A. Family B. School C. Travel D. Television

C. While travel can be highly educational in terms of introducing people to other cultures and ways of life, it is not specifically an agent of socialization; it does not necessarily help people learn how to relate, as one can travel alone and avoid contact with others (choice C is not an agent of socialization and is therefore correct). Family is one of the first and primary sources of socialization from which a person learns how to relate to others --- one's parents, siblings, and extended family (depending on the family structure). Lessons learned from the family often form the basis for many future relationship choices (choice A is an agent of socialization and is therefore wrong). School is another primary source of socialization, as teachers try to educate children about the values of the specific culture and the important skills needed to survive in that culture (choice B is an agent of socialization and is therefore wrong). Television, along with other forms of mass media, provide people with a glimpse of how to relate to the larger world, introducing people to other lifestyles as well as popular trends and ideas (choice D is also an agent of socialization and is therefore wrong).

What is the net volume of fresh air that enters the alveoli each minute, assuming that the breathing rate is 10 breaths/min, the tidal volume is 800 mL/breath, and the nonalveolar respiratory system volume (dead space) is 150 mL? 65 mL 95 mL 6500 mL 7850 mL

C: 6500 mL - tidal volume: amount of air entering the lungs in a single breath (800 mL/breath) - of this 800 mL, only 650 mL actually reach the alveoli per breath since 150 mL is dead space - net volume of air reaching alveoli: 650mL/breath X 10 breaths/min

Which one of the following is LEAST likely to be the iatrogenic result of diagnostic labeling? A. Eating disorders B. Drug addiction C. Social deviance D. Alzheimer's disease

D. Alzheimer's disease is caused by neurodegeneration in the brain. Accordingly, someone who has the plaques and tangles characteristic of Alzheimer's disease is likely to experience symptoms of the illness irrespective of whether his or her self-image is tied to the disorder (choice D is correct). The other three choices all have a strong conduct component inherent in the diagnosis. Accordingly, one who is labeled as having an eating disorder may be more likely to engage is disordered eating behaviors (choice B can be eliminated), while one who is labeled as a drug addict may be more likely to feel helpless to control or limit drug use (choice C can be eliminated). Someone labeled as a social deviant is much more likely to engage in deviant behavior because of this label (choice C can be eliminated).

Which of the following reflects the researchers' attempt to minimize adaptation effects during the experiment? A. Using wild mice rather than domesticated mice B. Using sugar water rather than pond water during the training stage C. Rearranging the bottles after a human subject detected sulfur D. Limiting the human subjects' inhalation time to two seconds

D. An adaptation effect is the phenomenon by which an organism becomes accustomed to a sensory stimulus, which results in a higher threshold for detection of that stimulus. Since the experiment in this study was designed to measure detection thresholds, it was necessary to limit participants' inhalation time to two seconds so that they would not become accustomed to the smell of sulfur. Otherwise, their olfactory thresholds for the detection of sulfur may have measured higher than their actual levels (choice D is correct; choices A, B, and C are wrong).

Assuming that Korean culture is androcentric, which of the following statements is most plausible? A. In Korea, women are considered strong and are therefore less susceptible to shin-byung. B. A woman given the diagnosis of shin-byung in the United States would receive the same social prestige as in Korea. C. Korean women have little motivation to seek out attention and prestige by claiming the symptoms of shin-byung. D. A Korean man who seeks help for shin-byung symptoms would most likely receive a different diagnosis.

D. An androcentric culture is one that is male-oriented; the male is considered the norm and is consistently preferred across the various aspects of the culture. As a result, anything that is related to females and the female experience is relegated to a lower tier of importance. If a male claimed to be experiencing symptoms characteristic of shin-byung, it is unlikely that he would be treated the same as a female. He would most likely receive a different diagnosis, particularly because the passage seems to indicate that this condition occurs only in Korean women (choice D is correct). In an androcentric culture, it is less likely that a woman would be seen as strong enough to resist this disorder (choice A is wrong). A culture that neither recognizes this culture-bound disorder nor rewards women for becoming shamans, such as the United States, is unlikely to provide the same social rewards for women with these symptoms (choice B is wrong). If there are limited avenues for women to gain social status in an androcentric society, there is a greater motivation for women to use unusual means in order to gain such status (choice C is wrong).

n a male individual with Down's Syndrome (trisomy 21), how many chromosomes would be visible at metaphase I of spermatogenesis? A. 23 B. 24 C. 46 D. 47

D. An individual with trisomy 21 has an extra copy of chromosome 21 (three total copies). During metaphase I, the developing gametes are still diploid (separation of homologues has not yet occurred), so this individual would have the normal 46 chromosomes plus the extra copy of chromosome 21, for a total of 47 chromosomes (choice D is correct; eliminate choices A, B and C).

Which of the following ranks the redox-active species of the electron transport chain in order of decreasing electron affinity? A. O2 > FAD > CoQ > NAD+ B. NAD+ > FAD > CoQ > O2 C. O2 > CoQ > NAD+ > FAD D. O2 > CoQ > FAD > NAD+

D. Because O2 is the terminal species in the electron transport chain (ETC), it must have the highest electron affinity and thus be first in the list (eliminating choice B). Since NADH donates its electrons to the first electron carrier in the ETC, the product of its oxidation, NAD+, would have the lowest electron affinity and would be last in the list (eliminating choice C). FADH2 enters the ETC by donating its electrons directly to ubiquinone (CoQ) so the oxidized FAD will have a lower electron affinity than CoQ (eliminating choice A).

ystolic blood pressure approaches 0 mm Hg when it reaches the: A. capillaries. B. arteries. C. aorta. D. right atrium.

D. Blood from the systemic circulation at the end of its circuit through the body enters the heart through the right atrium, and at this point its pressure is near 0 mm Hg (choice D is correct). Although blood pressure is relatively low in the capillaries, it is not 0 mm Hg, or it would never circulate all the way back to the heart (choice A is wrong). Blood leaves the heart through the aorta on its way to the systemic circulation and is at its highest pressure at that point (choice C is wrong); from the aorta the blood travels through many arteries, and the pressure in these vessels is still high (choice B is wrong).

he phosphoanhydride bond of ATP has a bond dissociation energy that is: A. negative, and the breaking of this bond will release energy. B. negative, and the breaking of this bond will require energy. C. positive, and the breaking of this bond will release energy. D. positive, and the breaking of this bond will require energy.

D. Bond dissociation energies are always given as positive values and are effectively defined as the potential energy of the separate atoms minus the potential energy of the bonded atoms. Bonded atoms are always at a lower energy state compared to separate atoms, which are defined as 0 energy. Therefore, bond energy is equal to 0 - (a negative quantity), which yields a positive quantity (eliminate choices A and B). Since energy is therefore always required to break a bond, the answer must be D.

The phenomenon of adjusting one's behavior based on the behavior of others is known as: A. social facilitation. B. obedience. C. motivation. D. conformity.

D. Conformity is a process whereby a person changes his or her behavior to be in line with another's behavior (choice D is correct). Social facilitation is a phenomenon whereby being part of group encourages people to work harder (choice A is wrong). Obedience differs from conformity in that the person changes his or her behavior to comply with the demands of an external authority (choice B is wrong). Motivation refers generally to all the possible reasons why people choose to act in particular ways (choice C is wrong).

Respiratory rate is regulated by many factors. Which of the following conditions is most likely to cause a decrease in breathing rate? A. Metabolic acidosis B. Low O2 concentration in the blood C. High CO2 levels in the blood D. High plasma pH

D. Consider the following equilibria: CO2 + H2O ?H2CO3 ? HCO3-. In the blood, CO2 from the tissues is quickly converted to carbonic acid, which then dissociates into hydrogen ions (lowering the pH) and bicarbonate ions. Excess CO2 thus promotes metabolic acidosis, which is quickly compensated for by increasing the respiratory rate. This helps to eliminate CO2 from the blood and restore pH to normal (choices A and C are wrong). Low O2 concentrations in the blood would lead to an increase in respiratory rate (choice B is wrong); furthermore, blood O2 levels affect the respiratory rate only in situations where O2 availability is low, such as at high altitudes (above 7000 feet). Of the choices given, the most likely condition to induce a decrease in respiratory rate is high plasma pH (choice D is correct) since a reduction in respiratory rate would lead to an accumulation of CO2 in the blood, and a corresponding increase in H+.

The higher than normal frequency of guevedoche in the Dominican Republic is the direct product of consanguineous relationships (relationships between close blood relatives). Which type of inheritance pattern is most consistent with increased phenotypic expression of a rare disease arising as a result of inbreeding within a population? A. X-linked dominant B. Autosomal dominant C. Mitochondrial inheritance D. Autosomal recessive

D. Dominant disorders are typically not rare (they have increased phenotypic expression) because they are caused by a dominant allele. An offspring need only receive one allele from an affected parent to express the disease. For example, suppose a man heterozygous for a dominant disorder (Aa) marries an unaffected woman (aa). The offspring all have a 50% probability of being affected by the disorder themselves. This is true for both autosomal disorders and X-linked disorders (choices A and B are wrong). Mitochondrial disorders are strictly maternally inherited, since the organelles of the zygote come only from the ovum. Thus, expression of this type of disorder does not change if a woman enters into a consanguineous relationship; the probability her offspring will inherit the disease depends only on her (choice C is wrong). However, recessive disorders are typically rare because they require a homozygous recessive genotype to be expressed, with one recessive allele coming from each of the parents. If mating is totally random, the frequency of a recessive disorder in a population is related to the frequency of the recessive allele; for example, if q = .001 (p = .999), then the frequency of affected individuals (qq) is only .000001, or 1 in 1 million. This number is the same from generation to generation, assuming mating stays random. Now consider what happens when mating is consanguineous. We'll use the same allele frequencies and look at two generations. Assume the first-generation parents were not consanguineous; they chose each other randomly. The probability of one of them being a carrier is 2pq. The probability of the father or the mother being a carrier is 2pq + 2pq, or 4pq. The probability that the carrier passes the recessive allele on to a son is 1/2, and the probability that the carrier passes the recessive allele on to a daughter is also 1/2. Now suppose the son and daughter mate and produce offspring. If they are both heterozygous, the probability of them having an affected child is 1/4. So the total probability of an affected child from a consanguineous relationship is: (probability original father is heterozygous + probability original mother is heterozygous) × probability the recessive allele is passed to a son × probability the recessive allele is passed to a daughter × probability son and daughter have an affected offspring = (2pq + 2pq) × 1/2 × 1/2 × 1/4 = 4pq/16 = pq/4 = .000999/4 ≈ 1/4000 This is a considerably greater probability than 1 in 1 million from the randomly mating population. Thus, consanguineous relationships (inbreeding) lead to increased expression of rare disorders. This is why this type of relationship is regulated legally, and is taboo in many cultures.

During the exponential phase of bacterial growth, bacteria are reproducing by: A. sexual reproduction. B. transduction. C. conjugation. D. binary fission.

D. During the exponential phase of bacterial growth, the population is increasing in size. The only choice which allows an increase in population size is choice D, binary fission. Sexual reproduction does not apply to bacteria (choice A is wrong), and while conjugation serves to increase genetic diversity, it does not increase the size of the population (choice C is wrong). Transduction is also a means to create genetic diversity. It occurs when a lysogenic virus excises from the genome and takes a portion of the genome with it. Any new hosts infected by that virus will be given the new genes, however this does not increase the size of the population (choice B is wrong).

Engagement rings are presented as betrothal gifts, often from men to their partners. These tokens would be of interest to which of the following? A. Conflict theorists B. Dramaturgical theorists C. Functionalists D. Interactionists

D. Engagement rings are presents to prospective spouses, but they are also important for another reason: engagement rings indicate that a person has accepted a marriage proposal. Interactionists are interested in micro-level interactions that create social meanings, thus focusing on communication through language and other important social symbols. Therefore, it is likely that interactionists would be interested in engagement rings because societies use these symbols to communicate that a person has made a formal agreement with his or her romantic partner and is now engaged to be married. These meanings help create our impression of the person wearing the ring, and individuals are thus active in shaping their societies, reflecting the arguments of interactionism. Furthermore, social interactions also regulate the practice of wearing the ring; for example, it is customary for engagement rings to be worn on the left ring finger in most Western countries (choice D is correct). The dramaturgical perspective is a specific form of interactionism that is interested in impression management (e.g., the back-stage and front-stage selves), but this is less applicable to the specific token presented upon engagement (choice B is wrong). The marriage-related interests of conflict theorists might be the internal power relations (or, to continue with the example, perhaps the relationship between socioeconomic status and the ranging qualities of the engagement ring, such as status); the pertinent interests of functionalists might be the contributions of marriage to social order. However, neither conflict theorists nor functionalists would be expected to consider the implications of an engagement ring in social interactions (choices A and C are wrong).

Which of the following represents the correct sequence for embryogenesis? A. Fertilization → gastrulation → blastulation → neural tube formation → somite formation B. Fertilization → gastrulation → blastulation → somite formation → neural tube formation C. Fertilization → blastulation → neural tube formation → gastrulation → somite formation D. Fertilization → blastulation → gastrulation → neural tube formation → somite formation

D. Fertilization is the first step, followed by a series of rapid cell cleavages to form a hollow ball of cells called the blastulam (eliminate choices A and B). Next comes the gastrula, in which cells move into the interior of embryo to form the three germ layers (eliminate choice C). Gastrulation is followed by the formation of the neural tube, which will form the nervous system, followed by the formation of other organs and tissues, such as the somites that will differentiate into bones and muscle. This makes choice D the only possible correct order of events.

What is the most stable conformation of trans-1-ethyl-3-isopropylcyclohexane? A. Both substituents are in the equatorial position. B. Both substituents are in the axial position. C. The isopropyl group is axial, and the ethyl group is equatorial. D. The ethyl group is axial, and the isopropyl group is equatorial.

D. In trans-1,3-disubstituted cyclohexanes, one substituent will always be axial and one equatorial; this eliminates choices A and B. It is more favorable (more stable) to have the larger substituent in the equatorial position. An ethyl group with two carbons is smaller than an isopropyl group with three carbons. The isopropyl group will therefore be equatorial and the ethyl group will be axial, so choice C can be eliminated.

Vinca alkaloids are a class of anti-cancer drugs derived from the periwinkle plant. Once absorbed into a cell, they interfere with the polymerization of microtubules. These drugs can prevent cancer from spreading by disrupting: A. pseudopod formation, thereby preventing cellular locomotion. B. prophase, thereby halting mitosis. C. transcription, thereby halting production of crucial cell proteins. D. metaphase, thereby halting tumor growth.

D. Formation of microtubules is crucial for formation of the metaphase plate. Microtubules polymerize from the centrioles outward. They contact centromeres (to become kinetochore fibers) and "push" the chromosomes towards the center of the cell to form the metaphase plate. Without proper microtubule polymerization, metaphase (and the rest of mitosis) cannot occur. If mitosis cannot occur, then neither can tumor growth. Note that microtubules are also required during prophase in forming the mitotic spindle; however, some parts of prophase can still occur in the absence of microtubule formation (DNA condensation, loss of the nuclear membrane). This makes choice D a better choice than choice B, since virtually all of metaphase depends on proper microtubule polymerization. Pseudopod formation requires the growth of microfilaments (actin fibers), not microtubules (choice A is wrong), and transcription (RNA polymerization) does not require microtubules at all (choice C is wrong).

Rogerian psychological theory, as developed by Carl Rogers, differs from Freudian theory in that: A. The Rogerian psychotherapeutic process focuses more upon early childhood events and conflicts than does the psychoanalytic process. B. Freudian theory emphasizes social influences, while Rogerian theory focuses on biological impulses. C. Freudian theory suggests that people are motivated by the need to become self-actualized, whereas Rogers posited that people are motivated to avoid anxiety. D. Freudian theory emphasizes innate biological drives and focuses on the conflict between the individual and societal demands, while Rogerian theory focuses on the individual's experience within the social environment and posits that individuals are motivated to become self-actualized.

D. Freud posited the existence of the id, which contains the biological impulses and unconscious desires, and emphasized that the superego and the ego formed in order to manage the individual's interactions with society. Carl Rogers also recognized the importance of the interaction of the individual with society, but, unlike Freud, posited that people are motivated by the desire to become actualized as the best possible versions of themselves (choice D is correct). Unlike Rogerian psychotherapy, the Freudian psychoanalytic process focuses largely upon the events and conflicts of early childhood (choice A is wrong). Freudian theory emphasizes biological, or innate, impulses, while Carl Rogers did not primarily emphasize biology (choice B is wrong). Rogerian theory (not Freudian theory) suggests that people are motivated by the need to become self-actualized, and Freud (not Rogers) posited that people are motivated to avoid anxiety (choice C is wrong).

2,4-dinitrophenol (2,4-DNP) is a highly toxic substance which was sold to the public as a weight loss drug in the 1930s. It acts by permeabilizing the inner mitochondrial membrane (IMM) to ions. Which of the following is true of 2,4-DNP's effects on oxidative phosphorylation? A. It causes decreased flux of electrons through ATP synthase. B. It likely leads to a decrease in body temperature. C. It leads to decreased consumption of FADH2 and NADH by the electron transport chain proteins. D. It causes a decrease in the electromotive potential built up by the electron transport chain.

D. If the inner mitochondrial membrane became permeable to ions, then hydrogen ions would not need to go through the ATP synthase in order to re-enter the matrix. This would dissipate the proton gradient established by the electron transport chain, and decrease its potential to generate ATP (choice D is correct). FADH2 and NADH would still be consumed by the transport chain proteins, since the electron transport chain is not shut down; it's just that the proton gradient would be more easily dissipated (choice C is wrong). The dissipation of the gradient would result in heat production; note that this is similar to what happens in brown fat, used by hibernating animals to stay warm (choice B is wrong). Electrons do not flow through the ATP synthase (choice A is wrong).

The researcher could NOT have discovered the rate law for the homogeneous reaction by measuring the change in total pressure in the reaction chamber if the reaction had: A. used a catalyst. B. taken place at room temperature. C. been allowed to proceed to completion. D. produced the same molar quantities of products as reactants.

D. If the reaction produced the same molar quantities of products as reactants, then there would have been no change in pressure, but pressure changes are precisely what determines the reaction rate for a homogeneous reaction involving gases.

An ectopic TSH-secreting tumor is suspected in a hyperthyroidic (high thyroid-hormone concentration) patient with goiter. Administration of a high dose of thyroid hormone confirms this, due to the fact that: A. [TSH] in the plasma decreases dramatically. B. [TRH] in the pituitary circulation increases. C. [TRH] in the pituitary circulation remains constant. D. [TSH] in the systemic circulation remains high

D. In a normal individual, high levels of thyroid hormone would suppress the release of TRH by the hypothalamus and TSH by the anterior pituitary. Injection of even more thyroid hormone should suppress the release of TRH and TSH even further. If this does not occur, and TRH and/or TSH levels remain high, it is likely that something outside the normal axis is involved. The passage states that ectopic tumors do not respond to normal feedback inhibition pathways; so, if an ectopic TSH-secreting tumor were present, the injection of thyroid hormone would not reduce the level of TSH in the system (choice D is correct and choice A is wrong). Since the tumor only secretes TSH, TRH levels should respond normally to the additional thyroid hormone—that is, they should go down due to negative feedback (choices B and C are wrong).

It was observed under the microscope that a small section of bone consisted of concentric rings of fibrous material surrounding an open circular region. The bone section was identified as an osteon. The circular region is expected to contain: A. mature osteocytes. B. maturing chondrocytes. C. large blood cell precursors. D. blood vessels.

D. In compact bone, the osteon is the basic unit of structure. Concentric rings surround the central circular region that contains blood or lymph vessels (choice D is correct). Osteocytes can be found in small spaces along channels branching away from the central channel (choice A is wrong). Bone is the site of the marrow in which blood cell precursors are found, but bone marrow is found in spongy bone and not in compact bone where osteons are found (choice C is wrong). Chondrocytes are found in cartilage (choice B is wrong).

The ion pumps for sodium and chlorine that establish the countercurrent multiplier system in the medulla of a vertebrate kidney are located in the cell membrane of the: A. proximal convoluted tubules. B. distal convoluted tubules. C. descending loops of Henle. D. ascending loops of Henle.

D. Ion pumps for Na+ and Cl- must exist in a part of the nephron that is permeable to those ions. As a result, answer C can be eliminated since the descending loop of Henle is permeable ONLY to water. Answer choices A and B can be eliminated based on knowledge of the anatomy of the nephron and kidney. The convoluted tubules are located in the cortex of the kidney. The question specifically asks "in the medulla of a vertebrate kidney." The loop of Henle is the only part of the nephron, aside from the collecting ducts, that delves into the medulla of the kidney. The deeper into the medulla the loop travels, the greater the countercurrent multiplier concentration that can be established, creating more highly concentrated urine.

Which of the following would impair nitrogen excretion? Increased glomerular filtration rate Impaired ammonia reabsorption in the proximal tubule Impaired glycolysis A. I only B. I and II only C. II and III only D. III only

D. Item I is false: increased glomerular filtration rate (GFR) would cause more urea to be excreted (choices A and B can be eliminated). Note that both remaining choices include Item III, so Item III must be true and we can focus only on Item II. Item II is false: if ammonia reabsorption is impaired, more ammonia will be lost in the urine (choice C can be eliminated and choice D is correct). Item III is in fact true: glycolysis is a primary pathway of pyruvate production. Since transport of ammonia from the muscle to the liver is pyruvate-dependent, reduced pyruvate production would impair ammonium excretion.

A patient presents with coronary artery disease (CAD) in which plaque has accumulated in the vessels, reducing but not blocking blood flow to cardiac muscle. What symptoms would be expected in this patient? Increased heart rate Incidences of chest pain Decreased oxygen supply to heart muscle A. I only B. II only C. I and III only D. II and III only

D. Item I is false: though plaque build-up in the coronary arteries is obstructive to those vessels, this does not have a direct impact on heart rate. A person with CAD could still have a normal pulse (choices A and C can be eliminated). Since both of the remaining answer choices include Item II, Item II must be true: chest pain is a common symptom of blocked coronary arteries, caused by a lack of oxygen delivery to the heart muscle. Item III is true: the supply of oxygenated blood would be limited (choice D is correct and choice B can be eliminated).

hirteen amino acids, including methionine, valine and proline, are glucogenic in humans. This means their α-keto acid carbon skeleton is converted to pyruvate during amino acid catabolism. After deamination, valine can therefore: Be converted into CO2 and H2O to generate ATP. Generate at least three NADH and two FADH2. Enter gluconeogenesis to generate glucose. A. I only B. III only C. I and II only D. I and III only

D. Item I is true: Amino acids are catabolized via deamination into α-keto acids and ammonia. Based on the information in the question stem, the α-keto acid formed from valine will be converted to pyruvate. Pyruvate can keep going through cellular respiration to generate CO2, H2O and ATP. Eliminate choice B. Item II is false: Pyruvate is converted into one acetyl-CoA (during which 1 NADH is made), and the acetyl-CoA would then generate three NADH and only one FADH2 as it cycles through the Krebs cycle. Eliminate choice C. Item III is true: Pyruvate can also enter gluconeogenesis to generate glucose. Eliminate choice A and choice D is correct.

Which of the following statements is/are accurate? Social identity involves an individual's self-identity. Social identity involves an individual's collective identity. Social identity can be a factor in in-group favoritism. A. II only B. I and III only Your Answer C. II and III only D. I, II, and III

D. Item I is true: social identity is an individual's self-perception as part of a social group. Accordingly, social identity involves one's self-identity (choices A and C can be eliminated). Item II is true: collective identity is the shared sense of belonging to a specific group, which certainly involves social identity and vice versa (choice B can be eliminated). Item III is true: in-group favoritism takes place when individuals value members of their own group more than those of other groups. Social identity allows these group distinctions to be made and therefore contributes to in-group favoritism (choice D is correct).

Marriage has distinct purposes in modern societies, and all of the following are considered manifest functions of marriage in most developed societies EXCEPT: A. the regulation of sexual behavior. B. the emphasis on monogamous relationships. C. the formation of lasting unions. D. the condemnation of adulterous actions.

D. Marriage, as a significant social institution, has multiple functions. The functionalist perspective distinguishes between manifest functions (those that are conscious and intended) and latent functions (those that are unconscious and unintended). The question stem specifies interest in the manifest functions - those that are made explicit through popular interactions and structures in societies. The condemnation of adulterous actions is better described as a latent function of marriage because it is simply incidental to society's promotion of the manifest marital functions (such as fostering lasting monogamous unions). Adultery is basically a violation of the marriage agreement to be monogamous, so it would not make logical sense for an intended purpose, or manifest function, of the institution of marriage to simply be condemnation for violating its own terms (choice D is not a manifest function and is the correct answer). The regulation of sexual behavior is a manifest function of marriage discussed in paragraph one of the passage. For example, it is common for societies to exalt the idea of "waiting until marriage" before engaging in sex (choice A is a manifest function and can be eliminated). The emphasis on monogamous relationships is also a manifest function of marriage in most industrialized societies. For example, monogamous relationships are considered the norm in the United States in that non-monogamous behavior is often considered deviant and sometimes even criminal; this expectation is reflected in Christian religious doctrines that exalt marriage between "a man and a woman" in the singular form (choice B is a manifest function and can be eliminated). The formation of lasting unions is perhaps the most obvious manifest function of marriage, even in light of the rising divorce rate (choice C is a manifest function and can be eliminated). Further functions of marriage include the definition of kin relationships (e.g., the relationships between parents and their children) and the regulation of property and other forms of wealth. Marriage is also the foundation of the highly important institution of the family.

Which of the following is NOT true of a magnetic field? A. It can be generated by a moving charge. B. It can accelerate a moving charge. C. It can exert a force on a moving charge. D. It can increase the speed of a moving charge.

D. Since the magnetic force FB on a charged particle q is always perpendicular to the velocity v of the particle—because FB = q(v × B)—the force FB can do no work on q. Therefore, by the work-energy theorem, W = ΔKE, FB cannot change the kinetic energy (or, therefore, the speed) of q. Thus, statement D is false.

The serotonin transporter (or SERT) removes serotonin from the synaptic cleft and recycles it back into the presynaptic cell. It thus terminates the effects of serotonin and this mechanism has been targeted in treatments for alcoholism, clinical depression, obsessive-compulsive disorder, and hypertension. SERT spans the plasma membrane 12 times and is also a: A. glycosylated phospholipid, with both hydrophobic and hydrophilic regions. B. peptide chain with at least four levels of protein structure, held together by disulphide and peptide bonds. C. protein with twelve hydrophobic domains, none of which contain the amino acid proline and at least some of which contain Beta-sheets stabilized by covalent bonds. D. protein with twelve transmembrane domains, each of which is an Alpha-helix with no proline and external hydrophobic residues, stabilized by hydrogen bonds

D. Membrane transport is mediated by proteins, not phospholipids (eliminate choice A). All proteins have at least three levels of protein structure, but only some have quaternary structure. There is no information in the question stem to support the fact that SERT contains more than one peptide chain (eliminate choice B). Transmembrane domains are alpha-helices with external hydrophobic residues. They cannot contain proline because of its secondary amine structure. Both alpha-helices and beta-sheets are stabilized by hydrogen bonds, not covalent bonds (eliminate choice C, choice D is correct).

Most biological unsaturated fatty acids are cis and contain non-conjugated double bonds. Because of this, additional steps are required in β-oxidation. These most likely include: A. Changing a single double bond to trans via a reductase enzyme in the mitochondrial matrix. B. Combining two double bonds via a reductase enzyme (which uses NAD+ as a reducing agent), then changing the resultant trans double bond to cis via an isomerase enzyme. Your Answer C. Moving a single double bond down the fatty acid chain via a translocase enzyme in the mitochondrial matrix. D. Combining two double bonds via a reductase enzyme (which uses NADPH as a reducing agent), then changing the resultant cis double bond to trans via an isomerase enzyme

D. Monounsaturated fatty acids require an isomerase enzyme to move the double bond during β -oxidation (eliminate choice A). Polyunsaturated fatty acids require both an isomerase and a reductase enzyme to complete β -oxidation; this also requires the reducing agent NADPH (choice D is correct). Note that NAD+ is an oxidizing agent (not a reducing agent) because it oxidizes another molecule, and is thus reduced itself (eliminate choice B). β -oxidation does not require a translocase enzyme (eliminate choice C).

All of the following biological developmental milestones that change the brain occur during the adolescent years EXCEPT: A. Cell proliferation (particularly in the limbic system and prefrontal lobes) B. Myelination C. Synaptic pruning (of unused or unnecessary connections) D. Codification of neural networks for basic motor skills

D. Neural network development for motor skills is part of early brain development and is essentially complete for basic motor skills (such as crawling, walking, jumping, and writing) before adolescence (choice D does not occur during adolescence and is therefore correct). Cell proliferation, particularly in the prefrontal lobes and in the limbic system, is one of the changes to the brain that occurs during adolescence, as the prefrontal cortex is required for abstract thinking (choice A is wrong). Myelination, the creation of myelin sheaths around neurons in the brain to strengthen connections, is another change that occurs during adolescence (choice B is wrong). Synaptic pruning, which clears the brain of unnecessary connections, is the third major developmental change in the brain during adolescence (choice C is wrong).

A person on a sled is moving to the right along a horizontal frictionless surface. The person throws a ball straight upward (as seen in the person's frame of reference). What will happen to the motion of the person and sled? A. They will slow down. B. They will speed up. C. They will speed up at first and then slow to their original speed when the ball lands. Your Answer D. They will stay at their original speed.

D. Newton's Third Law (or conservation of momentum) provides the solution. Before the throw, the person, sled, and ball all have horizontal velocity. When the person throws the ball upward in her reference frame, she exerts an upward force on the ball. Therefore a downward reaction force is exerted on her by the ball. This downward force cannot provide an acceleration in a horizontal direction (and indeed, it doesn't provide an acceleration in the vertical direction either because of the normal force from the ground on the sled), so there is no change in horizontal velocity or speed.

A child hits her brother and is punished by being sent to her room with no dinner. When she is about to strike her brother again the next day, she remembers how hungry she was the day before and stops herself. What kind of conditioning occurred here? A. Classical conditioning; her hunger is a biological reaction B. Classical conditioning; her hunger is involuntary C. Operant conditioning; thinking about hitting her brother makes her hungry D. Operant conditioning; her reaction is voluntary

D. One of the major differences between classical conditioning and operant conditioning is whether the change in behavior is voluntary or involuntary. In classical conditioning, a natural (unconditioned) stimulus gets paired with a conditioned stimulus so that the conditioned stimulus eventually causes the same type of reaction that the unconditioned stimulus does. In contrast, operant conditioning is about the subject's choosing to perform, or not to perform, a particular action because of the consequences that previously resulted from that action. In this case, the child is making a conscious decision not to hit her brother because of the negative punishment (lack of dinner) that occurred when she hit him yesterday; therefore, her parents are using operant conditioning to make her change her behavior (choice D is correct). Although the child's hunger is a biological reaction to not eating dinner, and is theoretically an unconditioned response, her parents are not attempting to make hunger a conditioned response, and therefore they are not engaging in classical conditioning (choice A is wrong). Although her hunger is involuntary, it is the child's decision not to hit her brother that is the focus, making the issue of whether or not her hunger reaction is voluntary irrelevant (choice B is wrong). If thinking about hitting her brother made her hungry, which it would not, this would be a situation involving classical conditioning, not operant conditioning (choice C is wrong).

Which of the following is NOT a cluster B personality disorder? A. Antisocial personality disorder B. Borderline personality disorder C. Histrionic personality disorder D. Paranoid personality disorder

D. Personality disorders are grouped based on similarities in their characteristics. Cluster B disorders are characterized by heightened emotional arousal, as well as by dramatic, erratic, or impulsive behavior. Paranoid personality disorder is characterized essentially by distrustful behavior; it is a cluster A (odd) disorder and not a cluster B (dramatic) disorder (choice D is not a cluster B disorder and is therefore correct). Antisocial, borderline, and histrionic personality disorders are all characterized by poor emotional regulation and are cluster B disorders (choices A, B, and C are wrong).

Which of the following neurochemical changes would increase the likelihood that a visual signal will be transmitted to the brain? A. An increase in post-synaptic glutamate released by the photoreceptors B. An efflux of Cl- ions from the photoreceptors C. An efflux of K+ ions from the bipolar cells D. Closure of Na+/Ca2+ cyclic nucleotide-gated (CNG) channels in the photoreceptors, which are normally open during rest (in the dark)

D. Photoreceptors are depolarized during rest (in the dark) due to the constant influx of positive ions through the CNG channels. Thus, in order to increase the likelihood that a visual signal will be transmitted to the brain (i.e., to make it more likely that a visual stimulus of light will be perceived as an image), these photoreceptors must be hyperpolarized. During rest (in the absence of light), photoreceptors are depolarized, constantly releasing glutamate, which inhibits the bipolar cells from depolarizing. For a visual signal to be transmitted to the brain, the photoreceptor must be hyperpolarized, which inhibits the release of glutamate, allowing the bipolar cells to depolarize. Closure of CNG channels in the photoreceptors would prevent positively-charged ions from continuing to enter the photoreceptor, and would therefore make the membrane potential more negative, or hyperpolarized; this would increase the likelihood that a visual signal will be transmitted to the brain (choice D is correct). An increase in post-synaptic glutamate released by the photoreceptors would decrease the likelihood that a visual signal will be transmitted to the brain because glutamate synapses on bipolar cells and inhibits them, as described in the first paragraph (choice A is wrong). An efflux of Cl- ions from the photoreceptors would further depolarize them; they would become more positively charged as a result of the loss of negatively charged Cl-, which would decrease the likelihood that a visual signal will be transmitted to the brain (choice B is wrong). An efflux of K+ ions from the bipolar cells would result in more negatively charged (or hyperpolarized) bipolar cells, making them less likely to fire, and decreasing the likelihood that a visual signal will be transmitted to the brain (choice C is wrong).

A proton gradient is most directly related to the functioning of: A. the Na+/K+ ATPase. B. the collecting ducts in the nephron. C. voltage-gated calcium channels. D. ATP synthase.

D. Proton gradients are established by the electron transport chain during aerobic respiration. The gradient is then used to power an ATP synthase (D is correct). The Na+/K+ ATPase hydrolyzes a molecule of ATP to move sodium and potassium against their concentration gradients (A is wrong), the collecting ducts in the nephron regulate osmotic balance of the body through interactions with ADH (B is wrong), and voltage-gated Ca2+ channels are regulated by voltage, not proton gradients (C is wrong).

Phalloidin is a poison which affects the depolymerization of actin. A cell treated with phalloidin would likely exhibit difficulties with: A. separation of chromosomes during mitosis. B. cell-cell adhesion. C. flagellum motility. D. motility via pseudopodia.

D. Pseudopodial mobility is coordinated by the polymerization and depolymerization of actin and would thus be affected by phalloidin poisoning (choice D is correct). Separation of chromosomes during mitosis is coordinated by the spindle apparatus, made of microtubules (choice A is wrong). Cell-cell adhesion is mainly coordinated by intermediate filaments and desmosomes (choice B is wrong). Flagella and cilia are microtubule-based (choice C is wrong).

Complete reduction of acetic acid will produce which of the following? A. O2(g) B. CO2(g) C. CH3CHO D. CH3CH2OH

D. Reduction of acetic acid will initially produce the aldehyde given as choice C, where the carbonyl carbon has been reduced from an oxidation state of +3 to +2. Choice D is the result of further reduction to an oxidation state of +1, while choices A and B do not correspond to the reduction of acetic acid.

If a bisexual female reports feeling social pressure to be either lesbian or straight, which causes her significant stress, what term best describes the stress this subject is feeling about her social role? A. Role conflict B. Master status C. Role exit D. Role strain

D. Role strain is when having a single status results in conflicting expectations; if a bisexual female reports feeling social pressure to be either lesbian or straight, which causes her significant stress, she is experiencing conflicting expectations about her bisexual identity (choice D is correct). Role conflict happens when there are conflicting societal expectations for multiple statuses held by the same person. For example, a male kindergarten teacher—societal expectations for being a man and being a kindergarten teacher easily come into conflict (choice A is wrong). Master status is when one part of someone's identity comes to dominate their identity, to the exclusion of other parts of their identity (choice B is wrong). Role exit is the process of disengaging from one role in order to take up another. An example is the process of disengaging from a "pre-med" identity to take on a "medical student" identity (choice C is wrong).

Protein folding involves both the formation of secondary and tertiary protein structure to ultimately generate a stereotyped final configuration. What is true of the folding process? A. A protein with a large number of hydrogen bonds is more likely to have a positive ΔHfolding B. Multiple equal-energy final structures are possible C. Buried polar amino acids increase stability D. The Gfolded is smaller than Gunfolded

D. Secondary and tertiary protein structure largely depends upon the formation of weak interactions including both hydrogen bonding and Van der Waals forces. Formation of these interactions will release energy resulting in a negative ΔH (choice A is wrong). The folding process occurs spontaneously in order to reach an energy minimum (choice D is correct) and having multiple potential final folded products would likely not result in a functional protein (choice B is wrong). Burial of polar amino acids in what is most frequently a nonpolar interior would significantly decrease the stability of the folded state (choice C is wrong).

The myelin sheath is a dielectric that surrounds axonal membranes and facilitates electrical transmissions of neuronal signals. Demyelination is implicated in the exacerbation of Alzheimer's symptoms. Impairment of cognitive abilities due to demyelination most likely occurs because: A. increased speed of processing results in improved episodic memory. B. decreased speed of processing results in improved semantic memory. C. decreased speed of processing results in improved episodic memory. D. decreased speed of processing results in impaired semantic memory.

D. Semantic memory refers to the memory of meanings and other concept-based knowledge. As demyelination occurs, neuronal signals that carry meanings slow down, damaging the memory of them (choice D is correct). Episodic memory is the ability to contextualize the memory of autobiographical events; increased speed of neuronal processing could result in improved episodic memory, but this would not impair cognitive abilities and is not what the question stem asks (choice A is wrong). Decreased speed of neuronal processing would not result in memory improvement, whether semantic or episodic (choices B and C are wrong).

A chemist prepares a 1 M solution of sulfuric acid. Which of the following gives the relative concentrations of the species in solution? A. [H2SO4] > [HSO4-] > [SO42-] B. [SO42-] > [HSO4-] > [H2SO4] C. [HSO4-] > [H2SO4] > [SO42-] D. [HSO4-] > [SO42-] > [H2SO4]

D. Since sulfuric acid is a strong acid and dissociation of the first proton is complete, undissociated H2SO4 will have the lowest relative concentration, eliminating choices A and C. Since the dissociation constant for the loss of the second proton is well below 1 (~ 10-2), HSO4- will exist in greater concentration than SO42-, making choice D the correct answer.

Solomon Asch is famous for his study on: A. obedience to authority. B. cognitive dissonance. C. norm violation. D. conformity.

D. Solomon Asch conducted the famous line study and others like it in order to study the effects of conformity. Asch presented subjects with pictures of three lines of varying lengths and asked them to match one of the lines to a line that he had on a separate card. Although one choice was clearly correct, when confederates in the room selected a patently incorrect choice the subjects often followed suit. Asch argued that this behavior was due to the pressure to conform (choice D is correct). Stanley Milgram and Philip Zimbardo are known for their experiments on obedience to authority (choice A is wrong). Leon Festinger and James Carlsmith are known for their cognitive dissonance research (choice B is wrong). Rob Cialdini is known for his work on norm violations (choice C is wrong).

All of the following contribute to speciation EXCEPT: A. geographic isolation of populations. B. genetic diversity. C. natural selection. D. maintenance of Hardy-Weinberg equilibrium.

D. Speciation requires two populations to become reproductively isolated so that they can no longer interbreed. Geographic isolation is the easiest way to allow two populations to diverge and become reproductively isolated (choice A is true and eliminated). To become reproductively isolated, the populations must diverge genetically, evolving differently, and this would require genetic diversity in the population to begin (choice B is true and eliminated). Natural selection could drive the changes in isolated populations that cause them to evolve into two species (choice C is true and eliminated). If Hardy-Weinberg is maintained, then there can be no changes in the allele frequencies in the gene pool of a population and no evolution, which would not allow speciation (choice D is false and the correct answer choice here).

How would the functionalist sociological perspective view chronic illness? The distribution of chronic diseases reflects social patterns in the distribution of wealth and power. Disease is a form of deviant behavior because it interrupts the contributions of people to their societies. The medical profession is important in regulating the production capabilities of those with chronic conditions. A. II only B. III only C. I and II only D. II and III only

D. Structural functionalism is centered on the concept of society as a living organism. According to this sociological model, each individual social structure has a distinct purpose that is important for the function of the greater society. Item I is false: health disparities as a result of social inequalities in the distribution of resources is of interest to conflict theorists, not to functionalists (choice C can be eliminated). Conflict theory is centered on the concept of social competition for limited resources, such as access to health care. Item II is true: deviance is defined as the violation of social standards of conduct. Because structural functionalism is concerned with contributions to the greater social structure, functionalists consider disease to be a cause of deviant behavior because it inhibits individual contributions, thus having a negative impact on social order (choice B can be eliminated). Item III is true: the importance of social maintenance in structural functionalism creates the need for sources of regulation, such as medical professionals with authoritative roles in health care (choice A can be eliminated and choice D is the correct answer). Functionalists would argue the importantce of medical professionals for two reasons. First, medical professionals can assist in the management of chronic illness, thus allowing individuals to continue making their productive contributions to society. Second, in the case of debilitating conditions, the structure of health care can create a separation between the acceptable and unacceptable. The behaviors of those unable to contribute to sustaining social order are regulated, and those with chronic illness are expected to conform to the expectations of medical professionals in order to maintain structure.

Retroviruses utilize reverse transcriptase to synthesize a complementary DNA copy from a genome composed of RNA. Telomerase functions as a type of reverse transcriptase as it synthesizes the protective telomere sequences on eukaryotic chromosomes. In order for telomerase to function, it must include: A. a DNA sequence. B. an RNA polymerase. C. RNA nucleotides. D. an RNA sequence.

D. Telomerase adds conserved repetitive sequences of DNA (telomeres) to the ends of eukaryotic chromosomes. The question states that it acts as a reverse transcriptase, thus it must create these DNA sequences from an RNA template. The template for these sequences is an RNA sequence included in the enzyme that is the complementary copy of the telomere being synthesized (choice D is correct and choice A is wrong). The enzyme is a DNA polymerase (it is synthesizing DNA) thus it does not include (or need) an RNA polymerase (choice B is wrong), nor would it need RNA nucleotides (choice C is wrong).

Which of the following is true regarding the function of the right and left ventricles? A. The right ventricle and left ventricle produce equal systolic pressures, but have unequal ejection fractions. B. The right ventricle and left ventricle produce both equal systolic pressures and equal ejection fractions. C. The right ventricle and left ventricle produce both unequal systolic pressures and equal ejection fractions. D. The right ventricle and left ventricle produce unequal systolic pressures, but have equal ejection fractions.

D. The 2x2 method can be used here. This method allows you to eliminate answer choices based on identifying that two answer choices have a similar structure with only a key word or two in difference. For example, looking at choices A and B, both choices indicate that both the left and right ventricles produce equal systolic pressures. We know that this cannot be true because the right ventricle only supplies the pulmonary circulation and would not require, nor have the ability, to produce the pressure needed to supply the systemic circulation, as the left ventricle does (choices A and B can be eliminated). Although the right and left ventricles produce unequal pressures, they do have the same stroke volume and therefore equal ejection fractions (choice A is wrong and choice D is correct).

If a drug inhibited the activity of the Na+/K+ ATPase in neurons, this would result in: A. increased cellular glucose uptake via secondary active transport. B. increased cellular glucose uptake via primary active transport. C. decreased cellular glucose uptake via primary active transport. D. decreased cellular glucose uptake via secondary active transport.

D. The Na+/K+ ATPase is used to generate a Na+ gradient which is then used for glucose uptake by cells via secondary active transport (choices B and C are wrong). Uptake of glucose is mediated by the Na+-glucose cotransporter. Inhibiting the Na+/K+ ATPase would destroy the Na+ gradient and in turn decrease glucose uptake (choice A is wrong and choice D is correct). Remember that secondary active transport does not directly use ATP, but instead relies on the gradient established by the direct use of ATP.

Margaret was more afraid of dying in a terrorist attack than dying from dangers lurking in her own neighborhood because she was watching so many news reports of deaths by terrorist attack. This is an example of what psychological construct? A. Normative influence B. Fixation C. Attribution theory D. The availability heuristic

D. The availability heuristic states that a person will make a decision based on the information most readily available in memory. If Margaret has been watching many reports of terrorist dangers on the news, she will be more likely to worry about those than the dangers lurking in her own neighborhood about which she has heard little (choice D is correct). Normative influence refers to a group effect that results from individuals' desire to be liked and to win the approval of others, which does not explain Margaret's fear of terrorist attacks (choice A is wrong). Fixation refers to a state wherein an individual is attached to things or behaviors more appropriate to an earlier stage of psychosexual developmental (choice B is wrong). Attribution theory describes a social-cognitive approach to creating cause-and-effect relationships. Informational influence means that she is using the information in an attempt to be correct. That is not her motivation; rather, she is creating cause-and-effect connections between the information she hears and the decisions she makes. She is not worried about being right (choice C is wrong).

Which of the following is a true statement about the diaphragm? A. It contains both skeletal and smooth muscle cells. B. Its effector neurotransmitters are norepinephrine and acetylcholine. C. It is innervated by the phrenic nerve and autonomic nervous system. D. It receives neural signals from the cerebral cortex and the brain stem.

D. The diaphragm is purely skeletal muscle (choice A is false) and as such, ACh is the only neurotransmitter used (choice B is false). It's innervated only by the phrenic nerve, not autonomic nerves (choice C is false). The phrenic nerve originates both in the cerebral cortex, for voluntary breathing, and in the brain stem for involuntary control (choice D is correct).

Lying is often employed as a means of persuasion. Which theoretical model is often used when attempting to determine why someone has been persuaded? A. Yerkes-Dodson B. Cannon-Bard C. Schachter-Singer D. Elaboration-Likelihood

D. The elaboration-likelihood model of persuasion attempts to explain the different ways individuals process stimuli, why they are used, and their outcomes on attitude change. According to the elaboration-likelihood model, there are two major routes to persuasion: the central route and the peripheral route. Under the central route, persuasion will likely result from a person's careful and thoughtful consideration of the persuasive argument; under the peripheral route, the person will focus more on superficial aspects of the argument (catchy-ness, flashy-ness) and spend less time engaging in deep thought about the actual content of the message, so persuasion will occur because of the superficial aspects of the message (choice D is correct). The Yerkes-Dodson Law suggests that performance increases with physiological or mental arousal, but only up to a point. When levels of arousal become too high, performance suffers, which is graphically illustrated as a bell-shaped curve that increases and then decreases with higher levels of arousal (choice A is wrong). The Cannon-Bard and Schachter-Singer theories both attempt to describe how emotion and physiological arousal are related (choices B and C are wrong).

A parallel-plate capacitor (with a vacuum in between the plates) is connected to a battery. In Step 1, the distance between the plates is increased while the battery is still attached. In Step 2, the battery is disconnected and a dielectric is placed in between the plates. Which of the following is true about the change of potential energy in each of the steps? A. ΔPE1 > 0 and ΔPE2 > 0 B. ΔPE1 > 0 and ΔPE2 < 0 C. ΔPE1 < 0 and ΔPE2 > 0 D. ΔPE1 < 0 and ΔPE2 < 0

D. The equation for capacitance is C = κε0A / d, where κ is the dielectric constant (κ > 1), ε0 is a constant, A is the area of the plates and d is the separation between plates. In Step 1, d increases, therefore C decreases. In Step 2, κ increases, therefore C increases. The potential energy stored in a capacitor can be written as (1/2)CV2 or Q2 / 2C. In Step 1, the battery remains attached which means that V remains constant, so use the equation for PE with the V term. In that case, PE ∝ C, so decreasing C means the potential energy must therefore decrease. This eliminates choices A and B. In Step 2, the battery is disconnected, which means that Q remains constant, so use the equation for PE with the Q term. In that case, PE ∝ 1 / C, so increasing C means the potential energy must therefore decrease. The correct answer is choice D.

Which of the following does NOT support Noam Chomsky's hypothesis that children are born with a universal grammar? A. Children do not mix up the order of subjects and verbs in their native language. B. Children tend to make mistakes conjugating irregular verbs (e.g., saying "I runned" instead of "I ran") when learning a language. C. Children in less stimulating environments develop language skills at approximately the same time as do children in more stimulating environments. D. Children tend to imitate their caregivers' speech patterns.

D. The fact that children will sometimes grow to talk similarly to the adults who raise them aligns with the behaviorist view that language is learned through modeling (along with classical and operant conditioning; choice D is correct). The fact that children do not make extreme grammatical errors, such as switching subject and verb order, even when they are first learning a language suggests that they may be born with an inherent ability to learn a grammar (choice A is wrong). Children who make errors of overregularization (i.e., applying standard rules to irregular situations) are using the grammar that they comprehend; they must actively learn something that does not fit this grammar (choice B is wrong). Since children in different environments show similar patterns of language acquisition, it may be that they all have a universal grammar through which they learn to speak, even when they are taught more or less than other children are (choice C is wrong).

The hippocampus is a subpart of which brain structure? A. Executive functioning B. Basal ganglia C. Occipital lobe D. Limbic system

D. The hippocampus is part of the limbic system of the brain; a system which is responsible for many aspects of human functioning, including emotions and memory (choice D is correct). Executive functioning is generally thought to be regulated by the frontal lobes, which control inhibition and attention (choice A is wrong). The basal ganglia is situated at the base of the forebrain and is mainly comprised of striatum, the globus pallidus, the substantia nigra, the nucleus accumbens, and the subthalamic nucleus. It is associated with voluntary movement, procedural learning, and emotions, but is not associated with episodic memory as the hippocampus is (choice B is wrong). The occipital lobe of the brain is at the back of the head, and is primarily associated with vision (choice C is wrong).

The iron law of oligarchy: A. asserts that, because of the constant struggle between the labor class and the ownership class, democracy will be inevitably replaced by socialism. B. explains how formal social norms are codified into laws. C. describes the process whereby tasks are broken down into component parts to be completed by different workers in an organization. D. suggests that democracy is not possible in large groups or organizations

D. The iron law of oligarchy is a theory in sociology that states that regardless of how democratic an organization is, all organizations will eventually and inevitably end up with a majority of the power in the hands of a few, thus leading to an oligarchy. The iron law of oligarchy suggests that democracy is practically and theoretically impossible, particularly in large and complex organizations (choice D is correct). Karl Marx asserted that, because of the constant struggle between the labor class and the ownership class, capitalism (not democracy) will be inevitably replaced by socialism (choice A is wrong), but this has nothing to do with the iron law of oligarchy. The iron law of oligarchy does not explain how formal social norms are codified into laws (choice B is wrong). Rationalization, which is a major component of bureaucracy, describes the process whereby tasks are broken down into component parts to be completed by different workers in an organization (choice C is wrong).

Which of the following is the most likely sequence of events in the infection of E. coli by phage T1? A. Replication of viral genome, production of viral DNA polymerase, translation of viral lysozyme, assembly of infectious virus B. Production of viral DNA polymerase, replication of viral genome, translation of viral lysozyme, assembly of infectious virus C. Translation of viral lysozyme, production of viral DNA polymerase, replication of viral genome, assembly of infectious virus D. Production of viral DNA polymerase, replication of viral genome, assembly of infectious virus, translation of viral lysozyme

D. The production of viral DNA polymerase must occur prior to replication of the viral genome since the polymerase is needed to do this (choice A is wrong). The last step must be production of lysozyme since this will lyse the cell, bursting it open and stopping any further activities required to produce infectious virus (choices A, B, and C are wrong). The correct events are in choice D: the expression of the viral DNA polymerase must come first to then replicate the genome. Infectious virus can then be assembled, and released by cell lysis induced by viral lysozyme.

f a teacher remembered a student's unusual name because, coincidentally, it was also the name of the street on which the teacher lived as a child, that teacher's ease in remembering would be an example of: A. explicit memory. B. social learning. C. depth of processing. D. the self-reference effect.

D. The self-reference effect describes how it is easier to remember that which is personally relevant or which can be linked to other memories (choice D is correct). Explicit memory is a type of memory in which one can "declare" and clearly articulate what one knows (choice A is wrong). Social learning is not about memory, but rather about learning through observing another and imitating his or her actions (choice B is wrong). Depth of processing concerns how deeply one thinks about the issues to be committed to memory. In other words, meaning is more easily remembered than form because one often ruminates more about the former (choice C is wrong).

Brain-imaging studies on deceit have shown that it leads to an alternate cognitive process when compared to truth-telling, which appears to be the default process of the human brain. In suppressing or inhibiting the truth, and often responding to the resultant stress, the brain becomes more active overall. Multiple areas are involved in the process, which are expected to include which of the following regions? The frontal lobes The limbic system The temporal lobes A. II only B. I and III only C. II and III only D. I, II, and III

D. The two main brain structures expected to be activated as a result of suppressing or inhibiting the truth are the cerebral cortex and the limbic system. The general functions of the cerebral cortex include perception, skeletal muscle movement, memory, attention, thought, language, and consciousness; the general functions of the limbic system include emotion, memory, and learning. The regions of the cerebral cortex expected to be involved in the process described in the question stem are the frontal lobes and the temporal lobes. Item I is true: the frontal lobes are involved in complex reasoning skills and problem-solving, both of which can occur in the process of suppressing or inhibiting the truth (choices A and C can be eliminated). Item II is true: the limbic system is involved in both memory retrieval, which allows the brain to compare the mental images (i.e., the truth versus the lie), and the resulting emotions related to deception (e.g., anxiety; choice B can be eliminated and choice D is the correct answer). Item III is true: the temporal lobes are involved in short-term memory, language comprehension, and emotion; consciously suppressing or inhibiting the truth suggests memory retrieval.

Stereotype threat is closely related to ultimate attribution errors. A white female is teaching high school social studies to a group of students who are racially and ethnically diverse. Which of the following statements is LEAST supported by the concept of the ultimate attribution error? A. A Hispanic male who was born and reared in California does poorly on a quiz about the United States, and the teacher assumes that he is unfamiliar with the country. B. A black male gets a 100% on a quiz about emperors, and the teacher thinks that he must have cheated. C. A Hispanic female frequently uses slang terms in her essays about North American geography, and the teacher attributes this to her being too lazy to learn proper grammar. D. A black female does poorly on a quiz about capitals, and the teacher assumes that she had been unable to adequately study the night before.

D. The ultimate attribution error occurs when negative behavior by a member of an out-group is attributed by an in-group member to the out-group member's characteristics (e.g., genetics, intelligence, personality, childhood background), while positive behavior by a member of an out-group is rationalized or explained away (e.g., good luck, special circumstances, etc.). Since the teacher explained the black female's poor performance in terms of a personal circumstance pertaining to that specific student, the ultimate attribution error was not made (choice D is least supported by the concept of the ultimate attribution error and is therefore correct). The teacher's assumption that the male Hispanic student is unfamiliar with the United States is based on negative stereotypes about Hispanics (choice A is wrong), as is the teacher's assumption that the female Hispanic student is too lazy to learn proper grammar (choice C is wrong). The teacher's assumption that the black male student only did well because he cheated is explaining away the individual student's good performance (choice B is wrong).

Because researchers chose to use a questionnaire, which of the following was missing from this study? A. Dependent variables B. Scientific method C. Independent variables D. A control group

D. The use of a questionnaire means that the researchers did not utilize a control group and thus did not have two groups participating simultaneously under different circumstances; this would simply not be possible with this methodology (choice D is correct). The dependent variables in this study are the preferences regarding age and gender of the students polled in the questionnaire—what the researchers were looking to study (choice A is wrong). The scientific method, in which hypotheses are empirically tested through the careful and systematic collection of relevant data. was used in this study and is ideally used in all scientific research (choice B is wrong). The independent variable here is the group of students polled (those between the ages of 18 and 30, identifying as either male or female; choice C is wrong).

If size exclusion chromatography (SEC) were used to separate a mixture of ethylene glycol oligomers, as shown below, with fractions further purified by gas chromatography (GC), what might be expected? This image is too complex to describe in a useful or meaningful way. A. The use of nonpolar solvents in SEC will lead to good separation and efficient purification by GC. B. The use of ether-type solvents in SEC will lead to poor separation, and subsequently to inefficient purification by GC. C. Oligomers with long retention times on the SEC column have long GC retention times. D. Oligomers with short retention times on the SEC column will have long GC retention times.

D. The use of one solvent or another shouldn't make a large difference in SEC retention times, since it's size, not polarity, that causes separation (eliminate choices A and B). Long retention times on the SEC column mean that the oligomers are small, which also means they are more volatile and have shorter retention times on GC. The converse is also true: short SEC retention times mean the oligomers are large and less volatile, with longer GC retention times, so choice C is false.

A 2-newton force is applied horizontally to a 1-kg mass as shown in the figure below. The mass moves with a constant speed of 10 m/s from point A to point B. The coefficient of friction, µk, equals 0.2. The figure shows a 2-newton force is applied horizontally to a 1-kg mass. The mass moves from point A to point B. The work done on the block is: A. a state function, because the work is adiabatic. B. a state function, because the work is not adiabatic. C. not a state function, because the work is adiabatic. D. not a state function, because the work is not adiabatic

D. The work done by the 2 N force and by the force of kinetic friction both depend on the (length of the) path taken by the sliding block. Therefore, the work done on the block is not a state function. Furthermore, since heat is lost due to friction, the process is not adiabatic.

An individual reports experiencing chronic stress because she cannot afford clothes suitable for job interviews. What does this individual lack as a result of her economic situation? A. Social capital B. Social reproduction C. Symbolic capital D. Cultural capital

D. This individual lacks a cultural asset that would increase her likelihood of attaining employment; cultural capital refers to certain non-financial assets that promote or allow upward mobility in society, such as education, social graces, proper speech, specialized attire, etc. (choice D is correct). Social capital refers to the benefits of belonging to social networks; the question stem mentions appropriate attire for job interviews rather than interpersonal connections (choice A is wrong). Social reproduction refers to the processes through which social, cultural, and financial capital are transmitted generationally; the question stem does not mention interaction between generations (choice B is wrong). Symbolic capital refers to certain intangible assets, such as prestige, honor, and recognition, that help one advance within a society (choice C is wrong).

1H NMR spectroscopy can provide all of the following information EXCEPT: A. the connectivity of atoms in a molecule. B. the number of nonequivalent hydrogens in a molecule. C. the chemical environment of the hydrogen atoms in a molecule. D. the splitting patterns of equivalent hydrogen atoms.

D. This question is asking for the FALSE statement. The number of signals in a 1H NMR spectrum indicates how many different types of hydrogens a molecule has (eliminate choice B). The splitting pattern of each signal provides insight into the connectivity of the atoms since nonequivalent neighboring hydrogens split the signals of their neighbors (eliminate choice A). The chemical shift of a signal generally indicates what types of groups are near the hydrogens in the molecule (eliminate choice C). While 1H NMR provides information about the splitting patterns of hydrogen atoms in a molecule, splitting effects only apply between nonequivalent hydrogens (not equivalent hydrogens). This answer choice is false, thus choice D is correct.

Helicobacter pylori, a bacterium found in the stomach and duodenum, has been implicated in the formation of peptic ulcers (lesions due to inflammation and low pH). Which of the following would be the most effective treatment to eliminate the infection and allow for healing? A. Solid antacids to neutralize the stomach acid B. Parietal cell agonist to kill the H. pylori C. Oral antibiotic and solid antacids D. Oral antibiotic and parietal cell antagonist

D. To treat peptic ulcers we must both eliminate the infection (so that the ulcer does not recur) and neutralize the stomach acid (to reduce the inflammation and allow healing). Treatment with the appropriate antibiotic will eliminate the infection (choices A and B are wrong) and blocking release of HCl by parietal cells with an antagonist will provide a sustained higher stomach pH than will solid antacids (either alone or in combination, choice D is better than choice C). Note that choice B might be particularly harmful; an agonist would increase acid secretion, the exact opposite of what we are trying to accomplish.

Which of the following helps to explain why a single candle burning in a dark room is very noticeable to the human eye, but a single candle burning in a well-lit room does not produce any noticeable difference in brightness? A. Top-down processing B. The law of figure-ground perception C. Parallel processing D. Weber's Law

D. Weber's Law states that the just-noticeable difference between two light stimuli is proportional to the magnitude of the stimuli - or, an increment is judged relative to the previous intensity of light. In other words, for the difference in two light stimuli to be perceptible, they must differ by a constant proportion, not a constant amount. This law explains why a lit candle in two different rooms (one dark, one well-lit) will be perceived quite differently by the human eye and brain (choice D is correct). Top-down processing explains how experience and expectations influence our visual perception, but does not explain why a candle in two different rooms produces brightness that is perceived differently (choice A is wrong). Gestalt laws of visual perception indicate that distinguishing the relationship between figure and ground is important. However, Gestalt laws of perception in general do not explain the difference in brightness of a single visual stimulus in two different settings (choice B is wrong). Parallel processing describes the brain's ability to simultaneously process incoming stimuli of differing quality, but this does not explain the difference in brightness of a single visual stimulius in two different settings (choice C is wrong).

Behavioral theorists explain which personality disorder as the result of having been rewarded for loyalty and punished for acts of independence? A. Histrionic personality disorder, which is characterized by excessive emotionality and attention-seeking behaviors B. Obsessive-compulsive personality disorder, which is characterized by perfectionism and a need for control at the expense of flexibility and efficiency C. Avoidant personality disorder, which is characterized by the avoidance of social interactions (despite a strong desire to be close to others) and an extreme sensitivity to negative evaluation D. Dependent personality disorder, which is characterized by a strong dependence on others to meet emotional and physical needs

D. With dependent personality disorder, individuals demonstrate patterns of clinging and obedience and fear separation, often insisting that they need to be taken care of and protected. Behavioral theorists believe that individuals with dependent personality disorder developed their disorder because of how they were treated by their parents when growing up; they were unintentionally rewarded for being clingy ("loyal") and were punished when they acted on their own (choice D is correct). Histrionic personality disorder, a disorder characterized by excessive emotionality and attention-seeking, is more often explained in terms of children having learned to behave dramatically in order to gain attention from distant parents (choice A is wrong). Obsessive-compulsive personality disorder may arise from struggles with parents over independence and control, but is not thought to involve reward and punishment (choice B is wrong). Behavioral theorists posit that individuals with avoidant personality disorder fail to develop normal social skills and come to expect rejection (choice C is wrong).

Psychological drug dependence is characterized by: A. tolerance, wherein higher and higher quantities of the drug are necessary to achieve the desired effect. B. downregulation of neurotransmitter activity in specific brain regions, such as the nucleus accumbens, in response to prolonged use of the drug. C. severe physical withdrawal symptoms upon cessation of drug use. D. the use or abuse of the drug in response to painful emotional triggers like depression or anxiety

Psychological drug dependence is characterized by the use or abuse of the drug in response to painful emotional triggers, such as depression or anxiety (choice D is correct). Psychological and physiological drug dependence are similar aspects of drug addiction; physiological dependence is marked by a physical "craving" for the drug, while psychological dependence is a "need" for a particular drug because it causes enjoyable mental effects (the two forms of dependence are often both present in drug addiction). However, psychological dependence is not characterized by tolerance (choice A is wrong), downregulation of neurotransmitter activity in the nucleus accumbens in response to prolonged use (choice B is wrong), or symptoms of withdrawal (choice C is wrong); these are all physiological phenomena associated with drug dependence.

Based on the passage, which of the following is LEAST likely to be a symptom of diabetes mellitus? A.Loss of appetite B.Sweet-tasting urine C.Unexplained weight loss D.Feelings of fatigue

The answer to this question is A, because based on the passage, the symptoms of diabetes mellitus are frequent urination and catabolism of fatty acid as well as proteins as alternative energy sources. In diabetic patients, high blood glucose results in excretion of excess sugar into the urine, hence sweet-tasting urine. Similarly, the catabolism of fatty acids and proteins results in weight loss and feelings of fatigue respectively. This leaves loss of appetite as the only correct answer

Which statement does NOT identify an aspect of the concept of assimilation? A.Assimilation is the influence that cultural changes have on an individual's health. B.Assimilation is the process of cultural adaptation that results from geographic mobility. C.Assimilation occurs when individuals adopt the cultural norms of a dominant culture. D.Assimilation occurs when individuals relinquish the cultural norms of their childhood.

The answer to this question is A. Assimilation is related to the process of social integration and generally refers to when new members adopt the main elements of a culture. The other response options each provide an aspect of the definition of assimilation (cultural adaptation, adopting new norms, and relinquishing old norms). However, the answer does NOT describe assimilation. Instead, it speculates about a possible consequence of assimilation.

Four organic compounds: 2-butanone, n-pentane, propanoic acid, and n-butanol, present as a mixture, are separated by column chromatography using silica gel with benzene as the eluent. What is the expected order of elution of these four organic compounds from first to last? A.n-Pentane → 2-butanone → n-butanol → propanoic acid B.n-Pentane → n-butanol → 2-butanone → propanoic acid C.Propanoic acid → n-butanol → 2-butanone → n-pentane D.Propanoic acid → 2-butanone → n-butanol → n-pentane

The answer to this question is A. The four compounds have comparable molecular weights, so the order of elution will depend on the polarity of the molecule. Since silica gel serves as the stationary phase for the experiment, increasing the polarity of the eluting molecule will increase its affinity for the stationary phase and increase the elution time (decreased Rf).

A certain bacterium was cultured for several generations in medium containing 15N, transferred to medium containing 14N, and allowed to complete two rounds of cell division. Given that the bacterium's genome mass is 5.4 fg when grown in 14N media and 5.5 fg when grown in 15N medium, individual bacteria with which of the following genome masses would most likely be isolated from this culture? A.5.4 fg only B.5.4 fg and 5.45 fg C.5.4 fg and 5.5 fg D.5.45 fg only

The answer to this question is B because DNA replication is semi-conservative. Therefore, after the first round of cell division the genome mass in each bacterium will be 5.45 fg (one DNA strand will contain 15N and the other strand 14N). Following the second round of cell division, half of the bacteria will have a genome mass of 5.4 fg (14N exclusively) and the other half a mass genome of 5.45 fg (14N in one DNA strand and 15N in the other).

Which research methodology involves the extended, systematic observation of a complete social environment? A.Comparative methods B.Ethnographic methods C.Experimental methods D.Survey methods

he answer to this question is B. Ethnographic methods are accurately described in the question as involving systematic observation of a complete social environment.

A researcher conducts observational research on the study habits of college students. When students are aware of the researcher's presence, they are more attentive, focused, and structured. When students are not aware of the researcher's presence, they are inattentive, unfocused, and distracted. Which concept best describes this phenomenon? A.Impression management B.The Hawthorne effect C.Self-fulfilling prophecy D.The Thomas theorem

he answer to this question is B. The Hawthorne effect describes changes in research participants as a result of their awareness that they are being observed. The changes to the students' study habits are best explained by the Hawthorne effect, which was first observed among workers. The incorrect options refer to related but distinct concepts. A self-fulfilling prophecy is an individual's internalization of a label that leads to a fulfillment of that label. Impression management refers to individuals actively managing how they are perceived by others. The Thomas theorem states that if an individual believes something to be real, then it is real in its consequences.


संबंधित स्टडी सेट्स

Chapter 11, 12 and 13 Nervous System

View Set

JA: UNIT 1: knowledge of capital markets

View Set

Chapter 27: Caring for Clients with Hypertension

View Set

Honors Geo Chap 5 Postulates, Theorems, & Definitions

View Set

Fiancial Management Ch. 12 Cost of Capital

View Set